Evidence Final

¡Supera tus tareas y exámenes ahora con Quizwiz!

Stan is charged with felony domestic violence after he allegedly struck his girlfriend in the chest with a baseball bat. At trial, the prosecutor will seek to offer evidence that six months earlier, Stan had beaten his former girlfriend with his fists so severely that she went to the hospital for two days. Stan is pleading self-defense, and at trial he will seek to admit evidence that three years earlier, his girlfriend got into a dispute with a store manager while she was shopping and she pushed the manager into a glass table, causing multiple lacerations and requiring the manager to get over twenty stitches. There is no evidence that Stan knew about the incident with the store manager at the time he struck his girlfriend with the baseball bat.

Both pieces of evidence are inadmissible regardless of the order in which they are offered

Raymond was on trial for vehicular manslaughter. The prosecutor argued that Raymond drove through a stop sign and struck a pedestrian, killing her instantly. Raymond acknowledges that he drove through the intersection and struck and killed the pedestrian, but he argues that the stop sign was covered up by an overgrown tree and so no reasonable person would have been able to see the sign. To prove his case, his attorney hires an investigator to go to the intersection six months after the accident and take photographs of the stop sign which show that it is blocked by tree branches. Raymond is present at the time and watches the investigator take the photos. At trial, Raymond wishes to admit the photographs as evidence. What procedure should he follow?

Call Raymond to the stand, show him the photos, and have him confirm that these photos are a fair and accurate representation of how the intersection and the stop sign appeared at the time of the accident.

The police pulled Maxwell over for speeding and got his consent to search his car. Inside his car, they found sixteen small baggies of crack cocaine. Maxwell was arrested and charged with possession of cocaine with intent to sell. Maxwell has made no statement at the time of his arrest regarding his intent, but during plea negotiations he admitted that he intended to sell the drugs, and he was willing to plead guilty and get a reduced sentence in exchange for giving information about his supplier. However, Maxwell ultimately backed out of his plea deal and the case went to trial. At trial, the prosecutor needs to prove that Maxwell intended to sell the cocaine instead of merely possessing it for his own personal use. What is the best way for the prosecutor to try to prove this fact?

Call a police officer with significant experience in the field of narcotics and qualify her as an expert. Then ask her if sixteen baggies is consistent with personal use.

In a civil case, Brent wants to admit a statement made by Carl as a statement against interest. Brent has already established that the statement was so contrary to Carl's pecuniary interest that a reasonable person would not have made the statement unless he believed it to be true. What else does Brent need to establish in order to admit Carl's statement under this exception?

Carl is now unavailable to testify.

Charles is on trial for reckless driving. He is guilty of this crime if he knowingly drove fifteen miles an hour or more over the speed limit; he is accused of driving sixty-one miles an hour in a forty-mile an hour zone. His defense is that his speedometer was broken, and that therefore he did not know how fast he was going. He testifies in his own defense and tells the court that he conducted an experiment the day after arrest to determine whether the speedometer was working. He told his brother to get in his car and said: "Drive next to me, and as soon as your speedometer hits sixty-one miles an hour, stick your hand out the window." Charles then drove next to his brother on the highway, and when his brother stuck his hand out the window, Charles looked down at his own speedometer and saw that it read fifty miles per hour. Which of the following statements is true?

Charles' brother sticking his hand out the window was hearsay.

Melinda is suing her former employer, Dollar Bank, claiming that her supervisor Reed sexually harassed her for months while she was working there. In its defense, Dollar Bank will call Reed to the stand. He will testify that he has supervised twenty different women over the course of the past three years, and that none of them have ever accused him of any inappropriate sexual behavior. This evidence is:

Inadmissible because of the bar on character evidence

Sarah's father was ailing and moved in to live with her. In order to provide him with the proper care, Sarah signed a contract with Diane Watson, a private nurse, to stop by her house every day to check on her father's health and give him medication. After two weeks, Sarah's father had died, and she believed that Watson's poor nursing care was partially to blame. She sued Watson, alleging that Watson failed to detect serious health problems and failed to administer the proper medication to her father. In her case-in-chief, Sarah wants to call Dr. Taylor, a physician who used to work with Watson. Dr. Taylor will testify that six months ago, Watson gave one of his patients the improper dosage of medication for his diabetes, causing the patient to go into insulin shock and die. Dr. Taylor's testimony is:

Inadmissible under Rule 404 because it is propensity evidence.

Hydra Energy signed a contract with Daystar Natural Gas, by which Daystar agreed to provide Hydra with 10 billion cubic feet of natural gas per year in exchange for a specified payment. After the first year, Hydra claimed that Daystar had only provided it with 9 billion cubic feet of natural gas. Daystar claimed that it had provided the required 10 billion cubic feet, and that Hydra's storage facilities were so poor that it lost 10% of the gas that it stored. In its case in chief, Hydra seeks to admit two court judgments against Daystar—one from two years ago and one from five years ago. In each case, a jury found that Daystar had failed to deliver the agreed upon amount of natural gas to an energy company with which it had signed a contract. Daystar objects to this evidence. The evidence is:

Inadmissible.

Taylor broke into a local convenience store and stole money from the cash register. When he got home, his wife Linda saw him carrying two plastic bags full of cash and asked him where the money came from. "I got desperate," Taylor said. "I stole it from the store down the block. Please promise you won't tell anyone." Linda was furious with him for committing the crime and she walked out on him and reported the crime to the police. Taylor and Linda are now divorced, and Taylor is on trial for burglary. The prosecutor calls Linda to the stand to testify about what she saw and what Taylor said on the night he came home with the money. Taylor objects. What is the proper ruling?

Linda can testify about what she saw, but she cannot testify about what Taylor said, because it is privileged.

Sven was charged with possession of illegal drugs with the intent to sell. As part of her case in chief, the prosecutor offered a lab report prepared by a chemist who worked for a private laboratory. The lab report confirmed that the substance which had been recovered from Sven was LSD, an illegal drug. The actual chemist who performed the test was not available to testify. Instead the prosecutor called another chemist who worked in the same laboratory to authenticate the lab report. The witness will testify that the recovered substances are routinely sent to the private lab from the police in order to conduct these tests. She will further testify that these reports are generated in the regular course of business of the lab, and that making the record was a regular practice of the lab, and that the lab report was made by someone with knowledge at the time he made it. Is the lab report admissible to prove that the substance recovered from Sven was in fact LSD?

No, because admitting the lab report would violate Sven's rights under the Confrontation Clause.

Garrett's dog bit Kyle in the leg. Kyle sued Garrett, seeking monetary damages. Under the applicable law, every dog owner is strictly liable for any injuries caused by his dog. At trial, Kyle offers evidence that Garrett's dog bit three other people in the year before biting Kyle. In his defense, Garrett offers evidence that Kyle kicked the dog just before the dog bit Kyle. Is either piece of evidence admissible?

No, because both pieces of evidence are irrelevant.

Barney is arrested for raping a 15-year-old girl. He refused to speak to the police and asked to be assigned an attorney. When he spoke to his attorney, he admitted that he raped the 15-year old girl and also told his attorney that he raped a 14-year-old girl two months ago, but that another man had been accused of that crime. The next day, Barney committed suicide in his cell. Barney's attorney investigated the case with the 14-year-old and learned that a man named Edgar is about to be tried for that crime. There is soem forensic evidence that corroborates Barney's claim that he raped the 14-year old girl. Can Barney's attorney testify in Edgar's trial about Barney's confession to that rape?

No, because the conversation between Barney and his attorney is privileged and Barney never waived the privilege.

Michael is suing Dakota Pharmaceutical. Dakota manufactures and sells an anti-depressant medication called Celodex, and Michael took Celodex for six months and then developed stomach cancer. He claims that the Celodex caused his cancer, and at trial he intends to call an expert witness to prove causation. In its defense Dakota intends to call its own expert witness, who testifies that Celodex cannot possibly cause stomach cancer. The judge also decides to call an expert witness of her own. In order to ensure the expert remains neutral, the judge prohibits either party from deposing her expert before trial, though she does give each party a copy of the experts' report. The report concludes that Celodex cannot cause stomach cancer. Michael objects to the judge's witness. Did the judge follow the proper procedure?

No, because the parties have the right to depose the court-appointed expert.

Norris is suing Stanley for breach of contract. Norris claims that Stanley sent him a letter, which Stanley signed, in which Stanley agreed to sell Norris a vintage comic book for $500. Stanley claims he never wrote any such letter and never accepted Norris' offer. Which of the following is a correct statement regarding the extent to which Norris can testify about the letter?

Norris cannot testify to the statement that Stanley made in the letter; he must have a photocopy of the letter or the original letter, and have it authenticated by someone who knows Stanley's signature.

Thomas is on trial for killing his sister Jane. His defense is that Jane was actually killed by Jane's ex-husband Max. Max has left the country, and is currently living in Brazil, where he is beyond the jurisdiction of the court. However, Thomas explains that Max felt guilty about Thomas being prosecuted for the crime, so he sent Thomas a letter in which he confessed to the crime. The letter is properly authenticated as having been written by Max. Thomas seeks to admit this letter into evidence in order to prove that Max, and not Thomas, is guilty of killing Jane. Is the letter admissible?

Perhaps. It is admissible to prove Max is the killer, but only if there are corroborating circumstances that indicate that the letter is trustworthy.

Which of the following would most likely be INADMISSIBLE?

Plaintiff is suing defendant for injuries sustained when the defendant hit her with his car. During settlement negotiations, the defendant admits that he may have been texting at the time of the accident, and he is willing to pay her half of what she is suing for to settle the case. Plaintiff refuses. During the defendant's case-in-chief, he testifies that he was not texting at the time of the accident. The plaintiff seeks to admit the statement defendant made during the settlement negotiations in order to impeach the defendant's testimony.

Which of the following is most likely to be admissible?

Plaintiff is suing the defendant in a civil suit alleging sexual harassment. The plaintiff alleges that the defendant, her supervisor, grabbed her buttocks and tried to kiss her in his office. The plaintiff offers the testimony of three women who will testify that when they worked for the defendant, he grabbed their buttocks or breasts in his office.

Woodlands Construction built a building for Ace Insurance Company. There was a dispute about the final cost of the project: Ace was contractually obligated to pay the full cost of all the building supplies, but Ace claimed that Woodlands was exaggerating the cost of the building supplies in its final invoice. Woodlands sent Ace four letters over the course of a month, demanding the money that it claimed was still owed and detailing the amount that it paid for the each of the supplies, but Ace refused to pay the disputed amount. Finally Woodlands sued Ace. At trial, Woodlands sought to introduce the letters it sent to Ace which described the amount it paid for each of the building supplies. Ace objected to these letters as hearsay. Should the court admit or preclude the letters?

Preclude the letters as hearsay.

George was loading a truck at work with boxes when the forklift operator accidentally hit him with the forklift, crushing his torso and breaking six ribs. George was rushed to the hospital and prepped for surgery to repair damage to his internal organs. As he was waiting to go into surgery, the Vice-President of the company he worked for came to him and said: "George, this was a terrible thing to happen. I want to assure you that the company will pay for this surgery and any rehabilitation that you need." George is now suing his company for negligent training and supervising of its employees, and he wants to testify as to the Vice-President's statement. Which rule of evidence would be the BEST one for the company to cite when it objects to this evidence?

Rule 409, which bars any offer to pay medical expenses.

Mortimer and Larry are both arrested for killing Mortimer's wife. The prosecutor's theory is that Mortimer wanted his wife dead so that he could inherit her money, and he hired Larry to kill her. After his arrest, Mortimer confesses to the crime, stating: "I needed my wife to be dead so I hired Larry to kill her for me." Larry makes no statements. The two defendants are tried together. Mortimer is not going to testify in the case. The prosecutor calls to the stand the police officer who interrogated Mortimer. The police officer is going to read Mortimer's confession to the jury. Larry objects. Out of the following options, which is the best action for the trial judge to take?

Rule that the statement will be precluded if hte two defendants are tried together because it is inadmissible against Larry and violates his Confrontation Clause rights, and the unfair prejudice against him cannot be cured with a limiting instruction.

Jarvis was driving home late at night when he struck Charles, who was on a bicycle. Charles was severely injured. The police were called and they gave Jarvis a Breathalyzer test. After he blew a .15—well over the legal limit—he was arrested and charged with driving while intoxicated. The next week Charles sued Jarvis, alleging that Jarvis was negligent, and seeking $100,000 in damages. One week before the criminal trial was set to begin, Jarvis offered to pay Charles $75,000 to settle the civil case. Will the prosecutor be able to admit the evidence of this offer in the criminal case against Jarvis?

Yes, if Jarvis told Charles that part of the conditions of the settlement would be that Charles refused to cooperate with the prosecutor in the criminal case.

Barry is on trial for possession of heroin with intent to sell. The prosecutor proves that Barry was caught in possession of sixty small baggies of heroin. The prosecutor asks the judge to take judicial notice of the fact that anyone with sixty bags of heroin intends to sell the heroin, because that amount is far more than anyone would personally use. Should the judge take judicial notice of this fact?

Yes, if the judge determines that this fact is beyond reasonable dispute and generally known to individuals in that area.

The state is prosecuting William for conspiracy to sell drugs. In its case-in-chief, the state offers the testimony of Raymond, one of William's former confederates who is now cooperating with the state. Raymond has a prior conviction for felony aggravated assault, which occurred eleven years ago. Raymond received probation for the crime. Can the defendant's attorney use that prior conviction to impeach Raymond?

Yes, if the trial judge finds that the probative value of the prior conviction substantially outweighs its prejudicial effect on the state.

Officer Jackson arrested William after William sold heroin to an undercover officer. Officer Jackson searched William pursuant to the arrest, and found a cell phone in William's pocket. The cell phone had a recent text message in plain view on the front: "How much stuff did you sell tonight?" Assume there are no Fourth Amendment problems with Officer Jackson reading the text. Officer Jackson now wants to testify at William's trial and tell the jury about the text message he read off the cell phone. Is Officer Jackson's testimony about the text message admissible?

Yes, it is admissible as evidence that William was selling things that night.

The prosecutor has charged Steven with aggravated assault on Andy. Steven is arguing self-defense, and Steven has an eyewitness named Sarah who was with Andy on the street a few minutes before the incident. Sarah will testify that she saw Andy pull out a knife and say: "I am going to slash Steven's face to teach him a lesson." Sarah then left the scene and saw nothing else. Should Sarah be allowed to testify in this case?

Yes, she can testify about seeing the knife and she can report what Andy said.

Carl is on trial for murder. He is pleading self-defense. The prosecutor has a character witness who will testify that in his opinion Carl is a vicious individual who routinely uses violence to get his way. Can the prosecutor's character witness testify at Carl's trial?

Yes, the character witness can testify, but only after the defendant calls his own character witness.

Blane is suing Taggert Industries for employment discrimination, alleging that Susan, Blane's supervisor and a Vice-President of the company, fired Blane because of his race. Blane calls Diedre as a witness. Diedre testifies on direct that she heard Susan, Blane's supervisor, using a racial epithet to refer to Blane a few weeks before Blane was fired. On cross-examination, Taggert's attorney asks Diedre if it is true that she told her friend a few weeks after the lawsuit was filed: "I am surprised; Susan has never done anything to make me think she was racist." Diedre denies having said this. Which of the following pieces of evidence (if any) could Taggert use to impeach Diedre?

Testimony from Diedre's friend that Diedre did in fact tell her "I am surprised; Susan has never done anything to make me think she was a racist."

Theresa is on trial for aggravated assault. She has testified in her defense. She has three prior convictions: one for aggravated assault (a felony) from six years ago, one for fraudulently filing a false police report (a misdemeanor) three years ago, and one for selling narcotics (a felony) one year ago. The prosecutor seeks to admit all three prior convictions as evidence in order to impeach her through proof of character for dishonesty. Which convictions (if any) are admissible?

The aggravated assault is almost certainly inadmissible; filing a false police report is certainly admissible, and the selling of narcotics will be admissible if the probative value of the evidence in proving the defendant's propensity for dishonesty outweighs its prejudicial effect on the defendant.

Police officers lawfully intercepted an e-mail sent from the account [email protected]. The e-mail offered to sell two hundred handguns, and it was sent to an undercover police officer. The prosecutor wants to authenticate this email as belonging to Grace Fell, who is being charged with illegal distribution of handguns. Which (if any) of the following pieces of evidence would likely be sufficient to authenticate the e-mail as having been written by Grace?

The e-mail account [email protected] is registered to Grace Fell and the content of the e-mail contains accurate, personal information about Grace, including her home address and a reference to her dog Butch.

The Internal Revenue Service ("IRS") began an audit of Jim's taxes. Jim hired Harriett, a lawyer, to represent him with his dealings with the IRS. Jim wrote Harriett an e-mail describing the problems with the IRS, and he attached the documents that had been prepared by his accountant when the accountant was calculating Jim's taxes. The e-mail was received by Harriet's paralegal Karl, who read the e-mail and then forwarded it to Harriett along with his (Karl's) own comments. The IRS now seeks to subpoena the documents prepared by the accountant, Jim's e-mail to Harriett, and Karl's comments that were attached to the e-mail when it was forwarded to Harriett. Which of these documents (if any) is privileged under federal law?

The e-mail and Karl's notes are privileged; the accountant's documents are not privileged

Vicky is suing her employer, Mershon Bank, for sexual harassment. She alleges that her immediate supervisor created a hostile work environment by making sexually explicit jokes in her presence and asking about her intimate relationships. In its defense, Mershon seeks to admit evidence that Vicky frequently told sexually explicit jokes at work and that she had a reputation as being sexually promiscuous with her co-workers. Is this evidence admissible?

The evidence about the sexually explicit jokes is probably admissible, but the evidence about her sexual reputation is not.

Debra was a fourteen-year-old girl who was being treated for severe depression. During her second session with her psychologist, she told him that her stepfather had sexually abused her, and she described the abuse in detail. This information was critical to determining how to treat Debra's depression. The stepfather was arrested and charged with child molestation. Debra refused to testify at the trial. The psychologist was called to the stand to testify about what Debra told her. The stepfather objects to this testimony as hearsay. How should the court rule on this exception?

The exception for medical treatment or diagnosis applies and the statement is admissible.

Which of the following facts would a court be most likely to take as judicially noticed?

The fact that Joe Biden was elected President of the United States in 2020.

Benjamin was a confidential informant working for the police. He wore a recording device to a meeting with Karl, a city official, posing as a developer that needed a variance on a zoning regulation. During the meeting Karl said that he would be willing to give Benjamin the variance in exchange for a cash bribe of $100,000. Benjamin took the recording device back to the police and played it for them. The police then made a transcript of the interview. Karl was arrested for soliciting bribes. Benjamin was called to testify about his meeting with Karl. Can Benjamin tell the jury what Karl said during the meeting?

Yes, Benjamin can testify as to the contents of the conversation between himself and Karl.

Oswald is accused of sexually assaulting his 17-year-old stepdaughter. The prosecutor calls a doctor to the stand to testify that she examined the stepdaughter one week after the alleged assault and found abrasions to the girl's genitals which are consistent with forcible intercourse. The defendant seeks to admit evidence that a few days before this incident, the stepdaughter had been sexually assaulted by a 16-year-old who was living with her in a foster home at the time. Is this evidence admissible?

Yes, but only if the abrasions could have been caused by the earlier incident.

The Securities and Exchange Commission ("SEC") is charging Delta Securities with insider trading. The SEC argues that over the course of three years, Delta made thousands of stock purchases and sales that profited off information that its employees had learned through improper means. At trial, the prosecutor seeks to introduce three graphs and five tables which summarize the thousands of transactions that the SEC had monitored over the past three years. Are the graphs and tables admissible?

Yes, but only if the prosecutor has made the underlying data available to Delta in advance of trial so that Delta can examine or copy the data. The underlying data must be admissible, but need not be admitted at trial.

Jenny and Harold met in a bar, and at the end of the night Harold offered to drive Jenny home. Instead of driving her home, he drove her to a friend's apartment and said that if she came inside he could get some marijuana from his friend and they could smoke it together. Jenny agreed. When the two of them got inside the apartment, however, their stories differ. Harold claims that his friend was not around, but that Jenny and he did smoke marijuana together and then had consensual intercourse. Jenny claims that there was no marijuana, and Harold forcibly raped her. Jenny went to the police with her version of the story the next day, and Harold was arrested and charged with rape. At trial, the prosecutor seeks to call Kathryn, who will testify that Harold raped her in the bathroom of a friend's house when they were at a party together. Kathryn never contacted the police about the alleged rape, so Harold was never charged. Harold seeks to call Stanley, who will testify that two weeks before Harold allegedly raped Jenny, Stanley and Jenny smoked marijuana together and then had sex. Should Kathryn and/or Stanley be allowed to testify?

Kathryn's testimony regarding Harold's alleged prior rape is admissible, subject to Rule 403, because it is evidence of specific act of sexual assault offered in a sexual assault prosecution. Stanley's testimony regarding his alleged sexual intercourse with Jenny is inadmissible because it is offered to prove propensity and violates the rape shield law.

Under which of the following cases (if any) would the attorney-client privilege apply?

Kathy is arrested for defrauding the federal government in order to receive higher Medicaid payments. She meets with her attorney and admits that she is guilty of the crime, but she wants to fight the case and take it to trial. Her attorney agrees and promises to do everything she can within the law to get the case dismissed or get a jury to acquit. The prosecutor now seeks to have the attorney testify about Kathy's confession, arguing that the crime-fraud exception to the attorney-client privilege should apply.

Which of the following out-of-court statements is most likely to be admissible in its entirety to prove the truth of the matter asserted?

"I am planning on going to school tomorrow to study geology."

George is on trial for assault. The prosecutor argues that George got in a fight with a man outside a bar and beat him severely. George testifies in his own defense, and tells the jury that he was not present at the bar; he was across town at home eating dinner at the time the beating occurred. George then calls a character witness. Which of the following statements by the character witness would most likely be admissible?

"I have known George for ten years and in my opinion he would never be violent against anyone."

Tracy is suing Windham Ladder Co., claiming that their ladder was manufactured with a design defect. She testifies on direct that the first time she used the ladder, it collapsed and she fell to the ground, breaking her ankle. Which question would most likely be permitted during cross-examination? (Assume that the opposing party has a good faith basis for believing that all of the facts being asked about are true).

"Isn't it true that you lied on your mortgage application when you tried to buy a house last year?"

Which of the following evidence (if any) is barred by the hearsay rule?

None of the above; all of this testimony would be permissible under the hearsay rule.

Zach is on trial for killing Penelope. He calls Randy as a witness. Randy testifies that he was once roommates with a man named Gerald, and that Gerald told Randy that Gerald had killed Penelope. The prosecutor wants to object to Randy's testimony. Zach claims that Gerald's confession was a statement against interest, and that he has already tried to subpoena Gerald to testify, but that Gerald refused to testify based on the Fifth Amendment. What is the prosecutor's best response?

"Randy's testimony does not qualify as a statement against interest, because Zach has offered no evidence to corroborate the testimony."

Cameron died in her bed five years ago. The police investigated the case as a possible homicide. An autopsy was performed by the county coroner and the cause of death was deemed to be an overdose of heroin. The police ultimately decided that the death was accidental and dropped their investigation. However, recently the police began to suspect that Cameron's death was not accidental, but that Mindy, his girlfriend, intentionally injected him with a fatal dose of heroin because she was angry with him for being unfaithful. Cameron's body has been cremated, and the coroner who performed the autopsy has died. The prosecutor is seeking to prove the cause of death, and she offers the autopsy report conducted by the county coroner. The coroner is a government employee. What is the best objection that Mindy's attorney can make to keep the autopsy report out of evidence?

"The autopsy report is hearsay and there is no hearsay exception which will allow it to be admitted."

Stephanie is suing Phillip for injuries she suffered when Phillip allegedly ran into her with a bicycle and then biked away. Phillip claims he was not even riding his bike that day and that someone else must have struck Stephanie with a bicycle. Stephanie calls Fran, an eyewitness, who will identify Phillip in court. In response, Phillip wishes to call Dr. Stevens to the stand. Dr. Stevens will testify that she is an experimental psychologist and has conducted dozens of tests and read about hundreds more on the topic of eyewitness identification. She will testify about the general unreliability of eyewitnesses, especially under stressful situations. She will also testify that she watched Fran testify and that based on her expertise Fran seemed to be unsure of her identification. What is the best objection that Stephanie can make to Dr. Stevens' testimony?

"Your Honor, Dr. Stevens' testimony will invade the province of the jury."

Edward and Rachel were married, but their marriage was running into difficulties. Once a week they met with Betty, a licensed social worker who provided them with marriage counseling. During these sessions, both Edward and Rachel talked about how their lack of money was affecting their relationship. After a number of sessions, Betty told Edward and Rachel that there was a way for them to get some extra money—if they claimed on their income tax returns to be making less money than they actually made, they would get a larger refund check during tax season. Betty showed them how to claim less income in a way that was likely to avoid detection. After the taxes were filed, the government detected the fraud and arrested Edward. Rachel fled the jurisdiction and is still at large. Edward is now charged with tax evasion. At trial, the prosecutor calls Betty to the stand to have her testify about the conversations she had with Edward and Betty about their tax returns. Edward objects, claiming therapist-patient privilege. What is the best response that the prosecutor can make to this objection?

"Your honor, there is an exception to the therapist-patient privilege if the therapist is assisting the patient in committing a crime."

Victor is suing John for breach of contract. During the trial, Victor called two witnesses, Wendy and Diane, who testified as to the fact that John did not deliver the goods as promised in the contract. John did not cross-examine Wendy at that trial. The trial ended in a mistrial when three of the jurors got the stomach flu during deliberations after the judge had dismissed the alternates. Thus, the case had to be re-tried. In planning for the re-trial, Victor decided not to call Diane to the stand because she had proved to be a poor witness. Thus, he intended on going forward only with Wendy's testimony. Unfortunately, on the day of the re-trial, Wendy did not show up to testify. Victor had not issued a subpoena for her, because he had believed she would show up voluntarily as she had for the prior trial. Victor did not want to obtain a subpoena on the day of the re-trial, because that would require getting a continuance and he wanted the trial to take place on that day. Therefore, Victor sought to admit Wendy's testimony from the prior trial. What is John's BEST objection when confronted with Wendy's testimony from the prior trial?

"Your honor, this witness is not unavailable, and so her former testimony is not admissible under Rule 804(b)(1)."

Which of the following opinions would most likely be admissible?

A narcotics detective with twelve years experience will testify that based on the amount of drugs recovered from the defendant and the way they were packaged, the defendant had an intent to sell the drugs.

Which of the following is probably not a sufficient authentication?

A police officer who took a blood sample from the defendant is shown the vial with the blood sample and he confirms that it is the same blood that he took from the defendant.

Sandy is arrested by the police for robbery. In which of the following situation is Sandy's confession LEAST likely to be admissible if offered by the prosecutor against Sandy? (Assume all Miranda rules have been complied with; i.e., there are no constitutional problems with any of the statements).

A few hours after the arrest, a police officer tells Sandy: "I just got off the phone with the prosecutor. She told me that she would be willing to charge you with a misdemeanor if you plead guilty at the arraignment and tell us right now the name of the person who committed this robbery with you." The police officer is lying—he has not spoken to the prosecutor—but there is no way for Sandy to know this. Sandy confesses to the robbery.

Which of the following is not self-authenticating?

A signed letter that is dated from twelve years ago, that transfers title of land.

George is suing Martha in a personal injury case, claiming that Martha's stairs were poorly maintained and therefore they crumbled when George stepped on them. One of George's witnesses is Frank, Martha's brother. During pre-trial discovery, Martha deposed Frank, and Frank said during the deposition: "About a week before George hurt himself, I saw that Martha's steps were rotting away. I told her to either get them fixed or to rope them off so that people wouldn't use them." George calls Frank at trial, but Frank refuses to testify against his sister, claiming "sibling privilege." The trial judge informs Frank that there is no such thing as sibling privilege, and that if he does not testify, he will be held in contempt. Frank still refuses to testify and is held in contempt and thrown in jail. George now seeks to admit Frank's statement during his deposition. Martha objects to the statement. How should the court rule?

AThe statement is admissible to prove the steps were rotting away and to prove Martha had notice they were rotting away.

Louis was pulled over for speeding by Officer Gareth of the Tarrytown Police Department. During this encounter, Officer Gareth allegedly ordered Louis out of the car and then struck him six times with his nightstick when Louis insulted him. Louis is now suing Officer Gareth and the Tarrytown Police Department, alleging that Officer Gareth used excessive force against him and that the Tarrytown police department failed to properly train, supervise, and discipline Officer Gareth. In his case-in-chief, Louis seeks to admit evidence that on three prior occasions in the past year, Officer Gareth had beaten motorists after he pulled them over for speeding. This evidence i

Admissible against the Tarrytown Police Department, but not admissible against Officer Gareth if offered to prove he has a propensity for violence

Jamie was opening a soda can when it exploded in her hands, causing serious injury. Her husband drove her to the hospital and dropped her off at the emergency room while he parked the car. She walked up to the desk and explained to the employee behind the desk that a soda can had exploded in her hands. Jamie later sued the company that manufactured the soda can, arguing that there was a defect in its design. During her case-in-chief, she called the hospital employee who admitted her to testify about what she (Jamie) said about the injury when she first arrived at the hospital. Jamie seeks to admit her statements to the employee to prove the truth of the matter asserted. Jamie's statements are:

Admissible if her statements were reasonably relevant to her diagnosis or treatment.

Harrison Oil runs ten large oil drilling rigs in the Gulf of Mexico. On July 12, a hurricane hit the region, and the high winds damaged one of the rigs, causing an explosion which killed two workers. The families of the workers are now suing Harrison Oil for wrongful death based on alleged negligent operation of the rigs. Harrison Oil kept the rigs running through the hurricane, even though the winds reached 85 miles per hour. One week after the hurricane, Harrison Oil changed its safety protocols so that it now shuts down its oil rigs anytime the winds get above hurricane force, around 74 miles per hour. During the plaintiffs' case-in-chief, the plaintiffs called an engineering expert who testified that the accident would not have occurred if Harrison Oil had shut their rig down before the hurricane and stored the more fragile equipment indoors. Another expert testified that the industry standard is to shut down oil rigs whenever winds reach hurricane force. During the defendant's case, Harrison Oil's Vice-President of Operations testified on direct that the company followed all necessary safety procedures during the hurricane and that the explosion was an unavoidable accident. He also testified that although it was common in the industry to shut down oil rigs during hurricanes, it would not make sense for Harrison to do so because it uses older model oil rigs which take six hours and dozens of employees to shut down the rig, which would endanger the employees' lives even more than if the rig were left running. The plaintiffs now wish to introduce evidence that Harrison Oil changed its safety protocols so that it now shuts down its oil rigs during hurricane force winds. This evidence will be:

Admissible only after the defendant's Vice-President has testified, and only to prove that it would be feasible for Harrison Oil to follow this procedure.

Julie Ann came home very late one night, and she was drunk, crying, and very upset. Her boyfriend Paul confronted her as soon as she got home, demanding to know where she had been. She started crying even harder, and explained that she was at a bar with some old college friends, and one of them, a man named Kyle, offered to drive her home at the end of the night. She agreed to go with him, but instead of taking her home, he drove her to an empty parking lot and raped her in his car. She said that he then drove her back to the bar and told her to get out of the car. She then walked ten blocks home. Kyle was arrested and charged with rape. At his trial, Julie Ann testified for the prosecution, and her story was consistent with what she told her boyfriend Paul on that night. The prosecutor then called Paul to the stand to report what Julie Ann told him when she got home. The defense attorney objects. Paul's testimony is:

Admissible under the excited utterance exception to the hearsay rule.

Oliver is an eyewitness to an armed robbery, and at trial he testifies for the prosecutor. On direct he testifies that he was about to enter his bank when he saw the defendant running out of the bank carrying a gun. He identifies the defendant in court. On cross-examination, the defense attorney asks Oliver if he ever filed a false insurance claim for his home insurance. This question is:

Admissible, as long as the defense attorney has a good faith basis for believing that Oliver has filed a false insurance claim.

After an argument about overtime pay, Cleon fired his secretary Fiona. Fiona called Cleon a few hours later and said that she had spoken to an attorney and that she intended to sue Cleon and the company for breach of contract. Cleon apologized over the phone and said that he had lost his temper. He then typed out a letter admitting that it was wrong to fire her, and offering to re-hire her and to pay her the overtime she had asked for. He then scanned the letter into his computer and sent her the letter attached to an e-mail. Fiona rejected his offer and sued Cleon, arguing that firing her and refusing to pay her overtime was a breach of their employment contract. She seeks to admit the letter as evidence that it was wrong for Cleon to fire her and that she deserved the overtime payments. The letter is:

Admissible.

Westin Coal owns and operates dozens of coal mines in the state. One day there was a cave-in on one of their mines, and three miners died. The miners' families are now suing Westin, arguing that the mines were negligently maintained. In their case-in-chief, the plaintiffs called a former Westin employee, who testified that no safety inspection had been conducted on the mine for over two years. During the defendant's case-in-chief, Westin called an employee from Prescott Insurance, a large national insurance company. The Prescott employee will testify that Westin has an insurance policy with Prescott, and that as part of that policy, Prescott employees conduct independent safety inspections on all of the Westin mines every three months. The Prescott employee will then authenticate and admit a business record from Prescott's files which confirms that Prescott conducted regular safety inspections up until the time of the cave-in. The plaintiffs object to the testimony of the Prescott employee. This evidence is:

Admissible.

Frank "The Snitch" Dunaway was arrested by an undercover officer when he tried to sell the officer five kilos of cocaine. As he was being arrested, Dunaway said: "You don't want me! You want my boss Victor! He gave me all these drugs to sell! I can tell you everything you need to know! Can we make a deal?" After Dunaway was arraigned, the officer took him to the prosecutor, where the prosecutor agreed to recommend probation if he pled guilty and testified against Victor. Dunaway then admitted to the crime and incriminated Victor in his statements. Later, pursuant to the agreement, Dunaway testified in front of the grand jury and admitted selling the drugs and incriminated Victor. When it came time to enter his plea, Dunaway refused to plead guilty, stating that he wanted to take the case to trial. At trial, the prosecutor sought to admit the following statements: (i) Dunaway's statements to the police officer. (ii) Dunaway's statements to the prosecutor. (iii) Dunaway's testimony in the grand jury. Should the court admit or preclude these statements?

Admit (i) and (iii), but preclude (ii).

Robert was driving his pickup truck alone when he swerved to avoid an animal in the road. His truck hit a tree, and he was thrown from the truck and through the windshield. He was pronounced dead on the scene. Robert's wife Linda is now suing the truck manufacturer, claiming that Robert had buckled his seat belt, but that the seat belt was faulty and it unlatched when the truck hit the tree, causing Robert to fly out of the truck. The truck manufacturer claims that the seat belt was not faulty, but that Robert had not put on his seat belt for this trip. Linda will testify that she has ridden with Robert over a hundred times in his pickup truck over the past few years, and that he always puts on his seat belt. The defendant truck manufacturer objects to this testimony. Should the court admit or preclude Linda's testimony?

Admit Linda's testimony as habit evidence.

Detective Starr works as a narcotics detective for the Jonesville Police Department. He has been indicted on charges of theft and cocaine possession with intent to distribute. The prosecutor alleges that six months ago Detective Starr arrested a suspect for cocaine possession, and found five kilograms of cocaine in the trunk of the suspect's car, but only turned in four kilograms to the police property room as evidence, keeping one kilo for himself in order to sell. Detective Starr denies the charges, arguing that there were only four kilograms of cocaine in the trunk originally and that he never stole any cocaine. At trial, Starr calls two character witnesses. The first is Detective Reynoso, Starr's partner. Detective Reynoso will testify that he has worked with Starr for five years and that Starr is the bravest police officer he has ever known. The second witness is Sarah, Detective Starr's girlfriend. Sarah will testify that she has known Starr for ten years and that he is extremely honest and law-abiding. The prosecutor objects to both witnesses. How should the court rule?

Admit Sarah's testimony but not Detective Reynoso's testimony.

Sam, Quentin, and Timothy were working together to sell cocaine. Timothy was arrested, and within a few hours he confessed, implicating Sam and Quentin. Sam and Quentin were arrested and charged with selling cocaine. Quentin was released on bail, and the next day he shot Timothy dead. He was caught in the act and pled guilty to the murder of Timothy and to the cocaine sales, stating that he and Sam had agreed that Timothy should be killed. However, he refused to testify at Sam's trial. Sam is now on trial. The prosecutor offers the statements that Timothy made during the confession which implicate Sam. Sam objects to this testimony. Should the court admit Timothy's statements?

Admit Timothy's statements, but only if the judge determines that Quentin killing Timothy was in furtherance of the conspiracy, within the scope of the conspiracy, and reasonably foreseeable as a natural consequence of the conspiracy.

Coby was involved in an altercation outside the bar, and police were called to the scene. The police officers attempted to arrest Coby for disorderly conduct, but Coby became more belligerent and began attacking the police officers. The officers responded with force, and ultimately Coby was thrown to the ground, where he hit his head. The injury ended up being fatal. Coby's wife is now suing the police department, alleging improper use of force and seeking monetary damages for the death of her husband. Under the applicable law, Coby's wife must show that the police were unreasonable in their use of force and that their unreasonable use of force caused Coby's death. If she proves liability, Coby's wife can recover for emotional loss and lost earning potential.At trial, the plaintiff called an expert witness to testify as to Coby's future earning potential. The defendant police department then sought to admit evidence that Coby had been convicted of three different felonies, including once for selling drugs and twice for aggravated assault. Defendant police department argues that this evidence is relevant to show that Coby's earning potential is not as high as the plaintiff's expert claimed. Coby's wife, the plaintiff, objects to this evidence. What is the judge's best course of action?

Admit the evidence but sanitize it by only allowing the jury to hear that Coby had three felony convictions, but not the names of the crimes.

Gary Smiles, a folk singer, is suing Amtrak Revival, a rock band, for copyright infringement. Smiles claims that Amtrak Revival stole the tune of one his songs "Forest Walk" and turned it into their number one hit "Burning Down the Forest." During the trial, Smiles wants to play his song "Forest Walk" for the jury and then play "Burning Down the Forest." In order for the jury to better hear the similarities, Smiles proposes that Forest Walk be played at twice the speed and in a slightly higher key than it was in the original, because then the similarities with "Burning Down the Forest" are more obvious. Amtrak Revival objects to this demonstration, arguing that instead Smiles should play "Forest Walk" at its normal speed and in its normal key. What should the judge do?

Allow Smiles to play "Forest Walk" at its normal speed and in its normal key and allow the jury to compare it with "Burning Down the Forest."

Sam is charged with breaking into a safe in his supervisor's office in June and stealing merchandise that his supervisor stored there. Only two people—the supervisor and the private security officer who installed the safe—supposedly knew the combination to the safe. At trial, the supervisor wishes to testify that he believes Sam broke into the safe in April, two months before the incident in question. He thinks this is true because he came to work early one day and saw Sam in his office for no reason, and then he checked his safe later that day and items had been moved around. Because nothing was stolen, he never reported this incident to the police. The prosecutor argues that Sam's alleged entry into the safe in April is admissible under Rule 404(b), because it shows that Sam had the knowledge required to break into the safe (either he somehow had the combination or was able to crack the safe); and the fact that he had this knowledge makes it more likely that he broke into the safe in June. Sam's attorney concedes that if Sam had broken into the safe in April, the supervisor should be able to testify to that fact, but he argues that there is insufficient evidence that Sam did in fact break into the safe. Should the court admit the evidence that Sam allegedly broke into the safe in April?

Admit the evidence if the judge believes a reasonable jury could find by a preponderance of the evidence that Sam committed the prior crime.

Gary and Charlotte went out on a date, and at the end of the date, they had sexual intercourse. Charlotte called the police the next morning and told them that Gary had raped her. When Gary was arrested, he claimed that the sex was consensual. Gary is now on trial for rape. Gary testifies that two nights before the alleged rape, he and Charlotte had gone out on a date which had ended in he and Charlotte kissing and engaging in other sexual activity short of intercourse. The prosecutor objects to this testimony. How should the judge rule?

Admit the evidence, because it is relevant to the case and there is an exception to the rape shield law for sexual conduct between the victim and the defendant.

Mark underwent routine surgery at Riverside Hospital. Complications from the surgery arose, and Mark sued the hospital and the surgeon who conducted the surgery. The case went to trial with both the hospital and the surgeon as co-defendants. ' At trial, Mark sought to admit a memo written by a hospital administrator in which the administrator admitted that the hospital made mistakes in prepping Mark for surgery. The judge decided that this memo was admissible against the hospital as a party-opponent statement, but inadmissible hearsay with regard to the surgeon. What should the judge do?

Admit the memo but give the jurors a limiting instruction telling them that they can only use the memo when determining whether the hospital is liable and that they should disregard the memo when determining whether the surgeon is liable.

Police had probable cause to believe that drugs were being stored in apartment 5D on the fifth floor of a certain building. They obtained a search warrant and searched the apartment, successfully finding a large quantity of heroin. However, there were no people in the apartment at the time, and the police were unable to determine who owned or lived in the apartment. The police then stationed themselves outside apartment 5D and waited. A teenager and his mother who lived in the building walked by and saw the police officers standing outside the apartment. The teenager turned to his mother and laughed, saying: "Those cops are morons. Big Jim isn't coming back to his place if there are a bunch of police waiting outside." A few minutes later an older woman walked by the apartment and the officer asked her: "Do you know where Big Jim lives?" and she pointed at the door of apartment 5D and then walked away. Police later arrested James Fanning for possession of heroin. At trial, the prosecutor established that in the neighborhood, Fanning is known as "Big Jim." The prosecutor then called the police officer to the stand, and asked him to testify about what the teenager said and what the older woman said. The evidence is being offered to prove that a person named "Big Jim" lived in apartment 5D. The defendant's attorney objected to this testimony as hearsay. How should the court rule?

Admit the statement by the teenager, but preclude the pointing by the older woman.

Louis is on trial for robbing a convenience store. The prosecutor seeks to call his girlfriend as a witness to testify that Louis was a regular user of heroin, and spent over $50 on heroin every day. She will also testify that Louis owed his heroin dealer over $500. Louis objects to this testimony as inadmissible character evidence. The judge should:

Admit the testimony about Louis' heroin use and his debt to the dealer to prove motive.

Miles is on trial for running a dog fighting ring. The prosecutor alleges that Miles owned a compound outside the city where he trained dozens of dogs to attack other dogs. Miles would then allegedly host large dog-fighting parties at the compound, where he would force his dogs to fight each other in a large pit and charge his guests to watch. In his defense, Miles calls his wife Janet to the stand. Janet testifies that she has known Miles for five years, and that at home he never showed any cruelty to animals, and was always kind to his own pets. Assume that the prosecutor has a witness named Sarah who was at Miles' and Janet's house a year ago and saw Miles kick his pet dog in the head after the dog did not obey his command. Sarah also reports that Janet was present and saw Miles kick the dog. How (if at all) can the prosecutor use this information at trial?

Ask Janet on cross-examination: "Isn't it true that you once saw Miles kick your pet dog in the head when the dog did not obey his commands?"

Dana is on trial for child abuse. The prosecution's theory is that on October 5th, Dana got angry at her six-year-old son for having stained the carpet in her bedroom, and so she struck him multiple times, causing significant bruising to his face and body. Which of the following evidence (if any) should be excluded as IRRELEVANT to the prosecutor's case?

All of the above facts are relevant to the case.

Denise and Linda were involved in an automobile accident, and Linda's car was destroyed. Denise was uninsured. At the scene of the accident, Denise apologized to Linda and wrote her a note which said: "I hereby state that this accident was my fault. I will pay whatever reasonable damages are due to Linda." A few days later, Linda sought payment from Denise. Denise refused to pay anything, saying she now thought the accident was Linda's fault. Linda sued Denise. At trial, Linda attempted to admit the note that Denise wrote at the scene in which Denise accepted responsibility for the accident. Denise denies ever having written the note. How can Linda prove that Denise wrote the note?

All of the above.

Frank and Allen ran a small software company together, and they developed a scheme to cheat on their taxes. They set up a dummy corporation, Alabaster, Inc., and they then made payments to Alabaster for "consulting" work. They deducted these payments from the reported income of their actual company and then paid taxes on the fraudulently lowered income. They then reported false "losses" for Alabaster so that Alabaster would have no income. Frank and Allen were ultimately arrested and their trial was severed. In Frank's trial, the prosecutor sought to admit an e-mail written by Allen to Frank which said: "I made another $50,000 payment to Alabaster today. That brings the total payments this year to $250,000." Frank objects to the e-mail as hearsay. Is Allen's statement admissible?

Allen's statement is admissible for the truth of the matter asserted as a party-opponent statement.

During a dispute in a bar, Charlie pulled out a knife and stabbed Harry in the chest. Harry died instantly. Charlie admits that he stabbed Harry in the chest, but he pleads self-defense, arguing that Harry swung at his face with a broken beer bottle first, and that he reasonably believed that he needed to use deadly force to protect himself. The prosecutor has a character witness who will say that Charlie has a reputation in the bar for being a very violent man. In his defense, Charlie wishes to testify that a few weeks before the incident, a friend of his had told him that Harry had stabbed two other people in the neck with a broken beer bottle in bar fights. How should the judge rule on the prosecutor's character witness and on Charlie's testimony?

Allow Charlie to testify about Harry's previous stabbing of two other people, but give a limiting instruction explaining that the jury should only use the testimony as evidence about Charlie's state of mind at the time of the stabbing, not as evidence of Harry's propensity for violence. The court should preclude the prosecutor from admitting evidence about Charlie's reputation for violence, either in the prosecutor's case-in-chief or in its rebuttal.

Esther and Ronald are married. Esther works as a maid for a wealthy family, and one night after Esther came home from work Ronald saw her saw her take a number of gold necklaces and bracelets out of her bag and hide them on the top shelf of a cabinet in the kitchen. One week later, Esther is arrested for stealing the jewelry. The police search the house, but by then Esther has sold the jewelry and so there is nothing to find. The police then interview Ronald and tell him that he should testify against Esther. Ronald agrees. Two months later, the prosecutor calls him to the stand to testify about what he saw on the night of the theft. Ronald is willing to testify, but Esther objects to the testimony, arguing that it is privileged. The two of them are still married at the time of the trial. Should the court allow Ronald to testify?

Allow Ronald to testify because he has waived the privilege.

Law enforcement officials searched Paul's computer pursuant to a warrant and found ten photographs containing child pornography. The officers seized Paul's computer and printed out copies of the images. Paul is now being prosecuted for possessing images of child pornography on his computer. At trial, the prosecutor does not admit the computer or the printed copies of the images. Instead, the prosecutor calls Officer Faraday, who conducted the search of Paul's computer and personally viewed the images. Officer Faraday will testify about the images of alleged child pornography that he saw on Paul's computer, describing the images in detail. What is the best objection that Paul can make to this testimony?

Best evidence rule

National Retail owns hundreds of warehouses across the country, where it stores merchandise for its discount retail stores. National Retail hired Cardez Shipping, a delivery company, to ship freight from its suppliers to its warehouses. After one year, National Retail began noticing that many of the boxes shipped by Cardez were arriving in the warehouses with significant damage. National Retail fired Cardez and sued the company for breach of contract under the Uniform Commercial Code. In its case-in-chief, National Retail called Zachary Winter, an industrial consultant who had worked for eleven years with companies in the shipping industry, advising them on how to run their businesses more efficiently. The Court certified Zachary as an expert in the shipping industry based on his education and experience. Zachary will testify to two points: (i) He will testify that he examined Cardez's quality control standards and that in his opinion they fell far below the industry standard for keeping freight safe and undamaged. (ii) He will further testify that this low level of quality control was a violation of at least three provisions of the Uniform Commercial Code, because his interpretation of the code was that, in the absence of any express contractual provision for quality control standards, the Uniform Commercial Code provided the applicable standard. Cardez objected to Zachary's testimony. How should the court rule on the admissibility of Zachary's testimony?

Allow Zachary to testify that Cardez's quality control standards were far below the industry standard, but do not allow him to testify that the Uniform Commercial Code applies and that Cardez's actions violate the Uniform Commercial Code, because that involves giving a legal opinion.

Gwen and her two children were flying on Patriot Airline, a charter plane company, when the plane crashed in the mountains. Gwen's husband sued Patriot Airline, alleging its pilot Ray was drunk when he was flying the plane and that he was improperly trained by his employer; thus, Gwen's husband alleges that the plane crashed because of pilot error. In its defense, Patriot Airline wants to call their Vice-President in charge of operations, who will testify about the extensive training and testing regimen that all of their pilots undergo before being entrusted to fly a plane. Patriot Airline also wants to admit evidence that the plane manufacturer who supplies planes to Patriot recalled its planes after the crash to fix an engine problem that tended to cause the motor to stall in cold mountain weather. Gwen objects to the Vice-President's testimony and to the testimony about the plane manufacturer's recall. Should the judge allow or preclude this evidence?

Allow both the Vice-President to testify, and allow evidence of the recall.

Sidney was charged with murdering his colleague Jake. The prosecutor's theory of the case is that Jake and Sidney had been embezzling money from the company for which they both worked. The prosecutor then alleges that Jake had been feeling remorse about the crime and told Sidney he was going to the police to tell them of the crimes. Sidney then allegedly killed Jake. At trial, the prosecutor seeks to admit evidence of the embezzlement that Sidney and Jake had been involved in. Sidney's attorney objects, arguing that Sidney is not charged with the embezzlement crime, so the jury should not hear about it. Should the judge allow or preclude evidence of the embezzlement?

Allow the evidence under Rule 404(b), because the evidence is not being offered to prove propensity to commit a crime but instead to prove Sidney's motive for committing the crime.

Damon is on trial for vehicular manslaughter after he allegedly killed a ten-year-old child while driving. The prosecutor has three witnesses. The father of the victim will testify that he saw Damon swerve off the road and up onto the lawn, where he struck the child. The police officer who responded to the scene will testify that Damon was crying and distraught a few minutes after the incident, and said that he only looked away from the road for a second to check his cell phone. A coroner will also testify that he examined the child's body, and that the cause of death was massive trauma to the chest, likely caused by a car. Damon objects to the father's testimony as unfairly prejudicial. He notes that the father will no doubt be very emotional when he testifies about his son's death, and that this extreme amount of emotion will unfairly influence the jury. Damon is willing to agree to admit that he swerved off the road, up onto the lawn, and struck the child. Damon says that once he has admitted this fact, the father's testimony has little extra probative value and should be precluded. The prosecutor still wants the father to testify. What should the court do?

Allow the father to testify with no limiting instruction.

Which of the following is likely to be INADMISSIBLE expert testimony?

An economist calculates the lost profit that a company suffered because of a trademark infringement. The economist relies in part on the statements made to him by the company president. The expert admits that economists almost never rely on statements made to them by corporate officers when making these estimates, but states that the business records alone were insufficient to draw a conclusion.

Who of the following would be LEAST likely to be allowed to testify?

An eyewitness who suffered brain damage and now has no independent recollection of the subject matter of the testimony. However, a few weeks ago the eyewitness read his own diary entries about the event that he made immediately following the event, and he is now willing to testify about what he read in his own diary.

Bernard was on trial on ten counts of conspiracy to sell narcotics and for arranging the murder of an undercover police officer. The chief investigator in the case, Detective Mums, has been investigating the case for six months and has helped the prosecutor prepare and coordinate the seventeen witnesses necessary to prove their case. The prosecutor tells the court that the detective is essential to her presentation. Detective Mums also plans on testifying in the case herself. Bernard intends to testify in his own defense. He also intends on calling his brother as a character witness. Before the trial begins, the judge issues a ruling under Rule 615 to sequester the witnesses. Who of the following is required to leave the courtroom?

Bernard's brother.

Which of the following questions is least likely to be objectionable under Rule 611?

Defense counsel asks an eyewitness: "Isn't it true that you and the defendant have never liked each other?"

Mark sued Suzanne in a contract dispute. As soon as Suzanne received a copy of the complaint in the mail, she made an appointment to see Amy, a lawyer specializing in contracts. The two of them spoke for over an hour, and Amy took detailed notes of their discussion. At the end of the hour, Suzanne hired Amy as her attorney in the case. A few months later, Mark made a discovery request for all of the notes that Amy made during the course of that initial discussion. How should the judge rule on the discovery request?

Deny the request because the information is protected by attorney-client privilege.

Sandra was working as a welder for Ace Welding. After one year on the job, she began to suffer from severe headaches and blurred vision. She is now suing Ace Welding, claiming that the company did not provide her with the necessary safety equipment on the job. In particular, Sandra claims that Ace did not provide her with a breathing mask that would filter out the poisonous fumes that resulted from welding metal. At trial, Sandra called Dr. Gunn, who had a PhD in material science and metallurgy. Dr. Gunn was qualified by the court as an expert in metallurgy. Sandra's attorney then asked Dr. Gunn two hypothetical questions: If an alloy of iron and copper were heated to the point at which they became soft and molten, would any fumes emanate from the heated metal? And if an individual breathed in these fumes, could it cause her to suffer headaches and blurred vision? Should the trial judge allow these questions?

Dr. Gunn should be allowed to answer the first question regarding the fumes that emanate from heated metal, but not about the second question about headaches and blurred vision.

Which of the following pieces of evidence is irrelevant under Rule 401?

During a political rally protesting police treatment of racial minorities, Donald allegedly broke a window of the police headquarters. Donald is now being prosecuted for criminal damaging. The elements of criminal damaging are: recklessly or knowingly causing damage to state property without permission or authority. Donald is Black. In Donald's defense, he wishes to present statistical evidence that the police use excessive force when arresting Black suspects at a rate three times more often than when they arrest white suspects.

Which of the following pieces of evidence is most likely to be admissible to impeach the witness?

Fred is on trial for burglary. He does not testify, but Victoria testifies for the defense, telling the jury that Fred was with her all night at the night of the burglary and so could not have committed the crime. The prosecutor seeks to admit Victoria's prior conviction for selling cocaine, a felony, from five years ago.

Alvin was a taxicab driver, and one night he drove his taxi off the road and onto the sidewalk, striking two pedestrians. One of the pedestrians broke his leg, and the other was seriously injured and was taken to the hospital. While the second pedestrian was in the hospital, Alvin's taxi company, Diamond Taxi, sent their attorney to talk to him. The attorney interviewed the pedestrian and took notes on what the pedestrian said, and in the margins the attorney also wrote his own thoughts about what other witnesses to find and possible defenses the company might have. The second pedestrian died the next day. The first pedestrian is now suing Diamond Taxi, alleging that Alvin was negligent in driving on the sidewalk. The plaintiff has made a motion to force Diamond Taxi to disclose the notes it made from its discussion with the second pedestrian in the hospital the day before he died. Diamond Taxi opposes the motion, arguing that the information is privileged. How should the judge rule?

Grant the motion for the notes on what the pedestrian said if the plaintiff can prove a substantial need for the information, but redact all of the notes made by the attorney on his own thoughts about other witnesses to call and trial strategy.

Greg was in prison, serving a lifetime sentence for rape. While in prison, he got into a fight with another inmate. Both inmates pulled out homemade knives, and Greg was stabbed in the chest. He fell to the floor and was bleeding profusely. As the guards rushed in to help, Greg grabbed one of them and said: "Listen. I don't have much time left. You should know that I killed a person three years before I was convicted of rape. The person's name was Charles Grundy, and he lived in Albany, New York." Greg then fell unconscious. The prison staff tended to his wound, but he died two days later. The guard investigated the case of Charles Grundy, and found out that indeed he had been murdered a few years ago. He also learned that a person named Kyle Worthington had been convicted of the murder, though Worthington always claimed he was innocent and that someone else committed the murder. A few months later, Worthington's conviction was overturned on appeal, and he was re-tried. The guard was called to testify at Worthington's new trial to tell the jury that Greg had confessed to the crime. The prosecutor argued that the confession was hearsay. Is Greg's confession a dying declaration?

Greg's confession is not admissible as a dying declaration because the content of the statement concerns a separate crime.

Fred is on trial for selling heroin. He has a prior conviction for burglary from two years ago. Under what conditions is it most likely that the prior conviction for burglary will be admitted in evidence?

If Fred testifies.

Greg is suing Veronica for breach of contract. Veronica has two prior convictions. Twelve years ago she was convicted of income tax fraud, a felony, and she served three years in prison. Six years ago she was convicted of filing a false police report, a misdemeanor, and she served six months in prison. Greg would like to admit the two prior convictions as evidence. Are they admissible?

If Veronica testifies, they are both automatically admissible.

Which of the following is permissible?

In a jury trial, the judge asks questions of one of the witnesses after the direct and cross-examination.

Jane Young, a six-year-old girl, was given antibiotics for a minor infection. Unfortunately, she was allergic to the antibiotics and she became very ill, spending over two weeks in the hospital. After Jane got ill, the doctor met with his two nurses and told them that they needed to conduct a basic allergy test on every new patient with an infection before the doctor could determine which antibiotics should be prescribed. Jane's parents sued the doctor who prescribed the antibiotics, alleging that he should have known that Jane was allergic to this specific strain of antibiotics and thus should not have prescribed them. The Young family had health insurance, so they did not have to pay any expenses aside from a $100 deductible for the hospital stay. However, they sued the doctor seeking medical expenses for the hospital stay (which would ultimately go to reimburse the insurance company, except for the $100 deductible) and for mental suffering (which the Young family would keep for themselves). At trial, Daniel Young, Jane's father, testified about his mental suffering, saying that there were many days when he did not know whether his daughter would live or die, and that he had to seek therapy in order to recover from the trauma. Daniel also testified that he suffered further mental anguish because "Hospitals are so expensive, and I would lie awake at night wondering how we were going to pay for her care and whether we would be able to afford the treatment she needed." Which of the following evidence is admissible?

In his case-in-chief, the doctor defendant seeks to admit evidence that the Young family was fully insured for the hospital stay and only paid a $100 deductible.

Gary and Thomas were arrested for conspiring to kidnap a six-year-old child in their neighborhood. A few hours after the arrest, Gary was taken to the prosecutor's office and the prosecutor read him his Miranda rights. The prosecutor talked with Gary for over an hour, and Gary ultimately agreed to plead guilty and testify against Thomas in exchange for a reduction in the charges against him. Gary later changed his mind and decided not to plead guilty. Thomas and Gary were tried separately for the conspiracy. Gary testified in his own trial, and Thomas also called him as a witness in Thomas' trial. When, if ever, can the prosecutor use the statements that Gary made to her after his arrest?

In the trial against Thomas, in order to impeach Gary if his testimony is inconsistent with his statements.

Granger Machines manufactures farm equipment and vehicles. John Kerling, a farmer, owned a harvester that had been manufactured by Granger. On January 5th, the harvester malfunctioned while Kerling was using it, and Kerling's arm was cut off. One week later, on January 12th, Granger sent out a recall letter telling all owners of the harvester to take them into the local Granger dealer, to add on a safety shunt that would prevent such accidents in the future. Daniel Sampson was another farmer who owned a Granger harvester. On January 13th, Sampson's hand was cut off by the harvester. Although the recall letter had been sent out at that point, Sampson had not yet received it. On January 15th, Kerling and Sampson each separately sued Granger Machines, alleging that the harvester had a design defect. In their respective trials, Kerling and Sampson each attempted to admit the recall letter sent out by Granger as evidence that the harvester was defective. Is the recall letter admissible for this purpose?

It is admissible in Sampson's case against Granger, but not Kerling's case against Granger.

Linda drank too much alcohol at a college fraternity party and passed out on a bed upstairs. While she was unconscious, someone came into the room and sexually assaulted her, although the perpetrator left no DNA evidence. After she woke up, Linda discovered what had happened and reported the incident to the police. The police conducted an investigation and ultimately arrested Daniel, one of the members of the fraternity that hosted the party, and charged Daniel with sexual assault. Daniel maintains his innocence, arguing that he never went upstairs at all during the party and never even saw Linda that night. The prosecutor wishes to call (1) George, one of Daniel's fraternity brothers, who will testify that he knows Daniel well and that Daniel has a reputation for taking advantage of women who had been drinking and making sexual advances towards women against their will; and (2) Jane, who met Daniel when she was drinking at a fraternity party six months ago, and who will testify that Daniel had fondled her breasts even though she repeatedly told him to stop and continuously tried to push him away. Should George and/or Jane be allowed to testify?

Jane should be allowed to testify about Daniel's prior sexual assault under Rule 413 (subject to Rule 403), but George's testimony is barred by Rule 404(a)'s ban on propensity evidence.

Nick is on trial for killing Stanley. Nick admits that he killed Stanley, but he argues that the crime was involuntary manslaughter and not murder. Murder is defined as knowingly killing a human being. Manslaughter is defined as knowingly killing a human being under extreme emotional distress brought on by a provocation that would cause a reasonable person to feel such distress. Nick testifies at trial. He says that thirty minutes before the killing, a friend of his named Nancy approached him while he was at a restaurant and told him that she had just come from his house. She further explained that she, Stanley, and Nick's wife Nadine had been together at the house using cocaine, that Nadine and Stanley had gotten into an argument, that Stanley had started beating Nadine with a chair leg, and that Nadine was beaten so badly that she was taken to the hospital. Nick immediately left the restaurant and ran to his house, where he saw Stanley talking to a police officer outside his house. Nick then took out a switchblade and stabbed Stanley in the chest, killing him. The prosecutor objects to Nick's testimony. She agrees that Nick can testify that Nancy came into the restaurant and said something, and he can testify that he left the restaurant and went to his house and stabbed Stanley. However, she argues that anything that Nancy told him is hearsay and should be precluded. Are Nancy's statements admissible?

Nancy's statements to Nick are admissible because they are not being offered to prove the truth of the matter asserted.

Officer Joiner pulled over a car with two people inside: Avery and Bart. When the officer looked inside the car, he saw a large bag full of white powder sitting in the back seat. He asked both Avery (the driver) and Bart (the passenger) if either of them owned the bag, and both of them said they did not own the bag. The officer seized the bag and performed a field test which revealed that the white powder was cocaine. Avery and Bart were both arrested and charged with possession of cocaine. Avery pled guilty, but Bart went to trial. At Bart's trial, the prosecutor called Officer Joiner to the stand and asked him to testify about finding the cocaine. The prosecutor then sought to qualify Officer Joiner as an expert in narcotics laws. Officer Joiner had been a narcotics detective for twelve years, he had taken a six month class in the laws of narcotics in his state, and he had testified in over a hundred cases. Once Officer Joiner was certified as an expert, the prosecutor asked him to explain the doctrine of constructive possession. Officer Joiner correctly explained that under the laws of the state, all the occupants of a car possess every item in the car that is not found on someone's specific person. Officer Joiner then gave his opinion that based on where the cocaine was found, it was constructively possessed by both Avery and Bart. Bart's attorney objects to the testimony. Was the testimony proper?

No, no expert can ever give an opinion about domestic law, since that is the exclusive domain of the trial judge.

Gary is on trial for robbing the First National Bank on December 10th. The prosecutor's theory is that Gary and his accomplice Bill robbed the bank together, wearing ski masks and driving away in the same car. Bill confessed to the robbery, and he has pled guilty and awaiting sentencing. He refuses to testify in Gary's case. The prosecutor seeks to admit Bill's confession. The confession does not mention Gary's name at all—Bill simply admits to having robbed the First National Bank with an accomplice on December 10th. The prosecutor has other evidence tying Gary to Bill—for example, Gary and Bill were together when they purchased ski masks the day before the robbery, and they were seen in a car together on the 10th only a few minutes before the robbery occurred. Gary's attorney objects to the confession. Is Bill's confession admissible?

No, the confession is not admissible because Gary has a Sixth Amendment right to cross-examine Bill and Bill is not available to be cross-examined.

Sandra and Bill made an oral contract in which Bill agreed to buy 12 different rare baseball cards from Sandra. The day after the two of them allegedly made the deal, Bill wrote down from memory the 12 different cards he had agreed to purchase as well as the purchase price for each card. The total purchase price according to his document was $5,280. The next day, Sandra delivered the cards and demanded $8,560. Bill protested, saying that was more than the price they had agreed on. Ultimately the two of them could not agree, and Sandra sued Bill for breach of contract. At trial, Bill testified about the deal he made with Sandra, but he could not remember exactly how much he had agreed to pay for each card. His attorney asked if he wrote down the prices for the cards. Bill said he did, and he confirmed that at the time he wrote down the prices, the prices were fresh in his memory and the record accurately reflected his memory at the time. His attorney then gave him the document he made the day after the deal was made, and (over Sandra's objection) Bill read it out to the jury. Bill's attorney then admitted the document into evidence (again over Sandra's objection). While the jury was deliberating, they asked to see the document, and (again over Sandra's objection) the document was sent back into the jury room for the jury to examine. Did the trial judge make the proper rulings?

No. Although it was correct to allow Bill to read the document to the jury, the document should not have been admitted into evidence or given to the jury.

Dana is on trial for killing her co-worker Harry. The prosecution's theory is that Harry and Dana went out on a date after work, and then went back together to Harry's apartment. At Harry's apartment, there was some type of argument, and Harry ended up dead with a stab wound in his chest. Dana concedes that she and Harry went out on a date after work, but claims that she left Harry after dinner and never went back to his apartment. The prosecutor wishes to call Detective Hansen, who will testify that Dana's fingerprints were found on a drinking glass in Harry's apartment. Detective Hansen will testify that he was on the scene looking for prints, found one on a drinking glass, and then sent the glass to Forensics-R-Us, a private, independent, forensic testing company. Detective Hansen also sent along a set of Dana's fingerprints, which he took from Dana as part of his investigation. Forensics-R-Us lifted the print from the glass and created a digital image of the glass fingerprint and of Dana's fingerprint and then isolated twenty different points of comparison on the glass fingerprint that were identical to twenty points of comparison on Dana's fingerprints. Forensics-R-Us sent the report to Detective Hansen. The Detective will testify that he read the report and concluded that the prints which were found on the drinking glass were Dana's. He will also testify that police detectives such as himself routinely rely upon outside forensic labs to lift the prints, process them, and compare them with the prints of known suspects. Is Detective Hansen's testimony admissible?

No. Detective Hansen cannot testify as to his opinion that the fingerprint at the scene matches Dana's print, nor may he explain the underlying data to the jury, because the underlying data is testimonial, and the individuals who created the underlying data are not available for cross-examination. Therefore, allowing Detective Hansen to testify would violate the Confrontation Clause.

Sam is on trial for raping Rosemary. Sam wishes to call Frank as one of his witnesses. Frank knows Rosemary well and used to date her. He will testify that he is familiar with Rosemary's reputation and that she is known as a woman who frequently engages in casual sexual intercourse. He will also testify that when he dated her, she was "sexually aggressive" and frequently initiated sex with him. Is Frank's testimony admissible?

No. Rule 412 precludes the defendant in a sexual assault case from offering any evidence about a victim's sexual predisposition.

Sam was riding his motorcycle when he was struck by George, who was driving a car. Sam sued both George and Tonda, the car manufacturer. Tonda reached a settlement with Sam, admitting that the brakes on the car were faulty as part of the settlement and paying Sam $100,000 in damages. George and Sam also had settlement negotiations, but they failed to reach an agreement, and the case went to trial. During the settlement negotiations, George admitted that he had drunk two glasses of wine that night before driving, but he claimed that he was still sober enough to see Sam and brake in plenty of time, and the car would have stopped if the brakes had been working properly. At trial, George took the stand in his own defense and testified that he had not had anything to drink that night and that the accident was due to the faulty brakes. At this point in the trial, which of the following is admissible?

None of the above.

Officer Farnsworth, Officer Young, and Officer Deason responded to a 911 call at a bar and found a man named Quincy throwing beer mugs and screaming at the other patrons. One of the beer mugs had hit another man in the head, and the man was bleeding severely. Officer Farnsworth began tending to the injured man, Officer Young approached Quincy, and Officer Deason stayed back to provide cover for Officer Young. Quincy had a broken beer bottle in his hand and was threatening Officer Young with it. Within a few seconds a struggle ensued, in which Officer Young wrestled Quincy to the ground and began kicking him in the head to subdue him. Quincy was ultimately arrested, and he suffered permanent brain damage as a result of the kicks. Quincy sued the police department, arguing that Officer Young used excessive force in apprehending him. The defendant police department called both Officer Farnsworth and Officer Deason as witnesses. Officer Deason testified that she observed the entire encounter, and in her opinion Officer Young used necessary force when he kicked Quincy, because Quincy was attempting to stand up and stab Officer Young with the beer bottle and the kicks were the only thing keeping him down. Officer Farnsworth testified that he did not see the encounter because he was tending to the wounded individual, but in response to a hypothetical question from the attorney, he testified that if Quincy were still trying to stand up and had a weapon in his hand, it would be necessary for Officer Young to kick him to keep him down. Neither Officer Farnsworth nor Officer Deason were qualified as experts by the court; they offered their testimony as lay opinions. Was their testimony admissible?

Officer Deason's testimony was admissible, but Officer Farnsworth's was not.

Which of the following questions to a witness is MOST likely to be allowed?

On direct examination: "Isn't it true that you are the owner of Stevens Garage on 415 S. Main Street?"

Martha is charged with defrauding a government contractor. The prosecutor has a character witness who will testify that he has worked with Martha for seven years and knows her reputation. He is willing to testify that Martha's reputation at work is that she is a very dishonest person. Under what conditions will the prosecutor's witness be allowed to testify?

Only after Martha testifies in her own defense OR calls her own character witness to testify that Martha is an honest person.

Sandy is on trial for selling crack cocaine. To prove the crime, the prosecutor will call Officer Grange, an undercover officer who will testify that he approached Sandy and that she gave him a rock of crack cocaine inside of a red balloon. Sandy argues that she was not the one who sold Officer Grange the cocaine—she was merely standing next to the seller, and the police arrested the wrong person. The prosecutor also wants to call Officer Hansen, an undercover officer who purchased crack cocaine from Sandy one year ago. Sandy was arrested for that crime, pled guilty, and served six months in jail. Officer Hansen will testify that when he bought the crack cocaine from Sandy one year ago, she gave him the cocaine inside a red balloon. Is Officer Hansen's testimony admissible?

Only if the judge is convinced that using red balloons to package crack cocaine is a distinctive and unusual practice for drug dealers.

Wilbur is a defense witness in a criminal case. After Wilbur testified, defense rested and the plaintiff called Emma, who works with Wilbur in the same company. Emma testifies that she has known Wilbur for six years and worked with him on numerous projects. Emma testifies as follows: Plaintiff's Attorney: What is your opinion of Wilbur's honesty? Emma: I would never trust Wilbur. I think he lies whenever he sees a benefit to it. He is certainly the type of person who would lie on the stand if he saw something in it for him. Plaintiff's Attorney: Can you tell me why you have this opinion?Emma: Sure. Well, there are a lot of reasons. The first time we worked together on a project, his part of the project was late, and so the entire project missed its deadline by a week. I learned a few months later that Wilbur had gone to our boss afterward and told him that the delay was my fault, even though my part of the project was on time. I was furious. Is Emma's testimony admissible?

Only in part. Emma should be allowed to make the first general statement about Wilbur's character, but she should not be permitted to describe the episode with the delayed project.

Sam is suing Super-Mart for employment discrimination, arguing that they refused to promote him because of his race. In his case-in-chief, he offers e-mails written by his supervisor at Super-Mart in which she uses a derogatory racial epithet to describe Sam. Super-Mart objects to the e-mail as improperly authenticated, and Sam's attorney does not respond to the objection. The e-mails are precluded. The jury ultimately finds Super-Mart not liable. On appeal, Sam argues that the e-mails should have been admitted, and he presents in his brief sufficient foundation to authenticate them. However, he never presented this foundation at trial. What should the appellate court do?

Overrule the lower court's ruling and remand for a new trial, with an order that the trial court reconsider the authentication issue in light of the evidence Sam's attorney has presented, but only if the preclusion of the e-mails affected Sam's substantial right and it was plain error for the judge to preclude them.

Harry is on trial for robbery. He calls Diane, a character witness, who testifies that she has known Harry for fifteen years and in her opinion he was an honest person who would never steal anything from anyone. On cross-examination, the prosecutor asks Diane: "Did you hear about the time that Harry was arrested for stealing a car two years ago?" Harry objects to the question. How should the judge rule on Harry's objection?

Overrule the objection and force Diane to answer. If the defense attorney requests, give a limiting instruction telling the jury that this is not meant to be evidence that Harry actually committed a prior theft, only a method of testing Diane's credibility and knowledge of Harry's character.

Harcourt Engineering had a contract with the federal government to build a number of high-energy power reactors. After three years and numerous cost overruns, the federal government sued Harcourt for fraudulently inflating the cost of the project. In order to prepare for the lawsuit, the general counsel of Harcourt sent a letter to the head engineer of the project, asking him to generate a list of all of the costs associated with the project The general counsel needed this information in order to prepare her defense, and the information was not otherwise available to the upper-level management of the company. The engineer complied with the general counsel's request. At trial, the government (the plaintiff in the case) called the head engineer to the stand and asked him to describe all of the costs associated with the project. Harcourt objected, citing attorney-client privilege. The court should:

Overrule the objection, because the attorney-client privilege only protects the communication itself, not the underlying facts contained in the communication.

Morris received a flyer in the mail from Able Home Improvement, offering a free estimate for evaluating his home's insulation needs. Morris called Able and asked Able to come out and give them an estimate. Able employees came to Morris' home and spent three hours in his attic inspecting his insulation. When they were done, they provided him with an estimate for new insulation and also a bill for $200 for the estimate. Morris protested, saying that Able's flyer had promised a free estimate. Unfortunately, Morris had thrown the flyer away the day after he called Able to come out, and his garbage had been taken away to the dump. Able demanded the money and ultimately sued Morris for non-payment. At trial, Morris sought to testify about Able's flyer which promised a free estimate. Able objected, arguing that the best evidence rule precluded the testimony. How should the judge rule on the objection?

Overrule the objection, because the original has been lost.

The First Union Bank was robbed on September 1. The robbers fled in a car that was seen driving into an area of the city known as the Downs. Officers found the car, abandoned, two hours later in the Downs. The detective in charge of the case reviewed the footage from the bank's security cameras and found one second of film in which one of the perpetrator's faces was uncovered and visible. The detective made a still photo from this one second of footage and then gave it to dozens of officers, who went door to door through the Downs, telling people that they were investigating a crime and asking them if they recognized the photo. Officer Brown was carrying out this assignment when he knocked on the door of Beverly Grogan. He showed her the photo and she gasped, putting her hand to her mouth and saying: "My God, that's my husband!" She then started crying. Officer Brown reported this to the detective, and her husband was ultimately arrested for robbing the bank. At trial, the prosecutor called Officer Brown to the stand and asked him what Beverly Grogan said when he showed her the photo of the bank robber in order to prove that the defendant was the person who robbed the bank. The defendant objects. What is the proper ruling on defendant's objection?

Overrule the objection, because the statement is an excited utterance.

James and Mike were stockbrokers who worked together at the same brokerage firm. James is now on trial for insider trading. The prosecutor wants to admit statements made by Mike which implicate James in the insider trading scheme. James objects, arguing that Mike's statements are hearsay. The prosecutor responds that although Mike's statements are hearsay, they should be admitted as an opposing party's statement because Mike and James were co-conspirators. The trial judge rules that Mike and James were co-conspirators and admits the evidence. James appeals the trial court's decision to admit Mike's statements. How should the appellate court rule?

Overturn the lower court ruling if the ruling was an abuse of discretion and affected James' substantial rights.

Which of the following opinion testimony would most likely require an expert witness in order to be admissible?

Percy was a witness to a vicious beating during which three police officers injured a suspect by kicking him and hitting him with nightsticks as he lay on the ground. Percy will testify that given the number of kicks and the severity of the blows, the suspect almost certainly suffered multiple broken ribs and permanent damage to his lungs and other internal organs.

Greg rented a stand-alone unit from Store-Your-Stuff storage facility in Florida, and he put six boxes containing his rare book collection inside the unit. Two weeks later, a severe rainstorm hit the area, bringing over 3 inches of rain. Because the roofs of the storage units had not been replaced for over twenty years, they could not keep out such a large quantity of rain, and water leaked into Greg's unit, destroying his books. Greg is now suing Store-Your-Stuff, arguing that the facility was negligent in not replacing the roof or taking any other action to prevent leaks. After some investigation, Greg learned that one week after the storm, Store-Your-Stuff installed tarps over all of its units. At trial during his case-in-chief, Greg wants to admit the fact that Store-Your-Stuff put tarps on the roofs of the storage units. Store-Your-Stuff has not yet contested that attaching a tarp to every roof would not be feasible, though Greg suspects that they might do so when they present their defense. How should the court rule on the evidence of the tarps?

Preclude all evidence of the tarp installation in Greg's case-in-chief.

Chrome Motor Company developed a new type of SUV called the Growler. Unfortunately, within two weeks after the Growler was out on the market, three different owners reported problems with the accelerator pedal. In all three cases, the driver reported that the accelerator pedal stuck, causing the car to accelerate uncontrollably, resulting in an accident that caused injury and property damages. All three owners sued Chrome, alleging that the Growler had an unsafe design that caused the accelerator to stick. Chrome settled with the first plaintiff for $50,000 and the second plaintiff for $100,000. However, the third plaintiff rejected all settlement offers and took the case to trial. At trial, the attorney for the third plaintiff sought to admit the fact that Chrome settled the other two cases, as well as the amount of money Chrome paid for the settlement. The attorney argued that these settlements proved that Chrome acknowledged the design was faulty. Chrome objected to this evidence. Should the judge admit or preclude the evidence of the settlements?

Preclude any evidence from either settlement.

Frank began digging a large hole in his backyard to install a small pond when he cut through an underground cable belonging to Starr Telephone. A Starr employee immediately came out to Frank's house to assess the damage, and he told Frank that the broken cable provided phone service to hundreds of businesses in the area, and having it broken was costing the company $10,000 per hour. It took the employee over four hours to fix the damaged cable. Two days later Frank got a letter from an attorney in Starr's office, demanding that he pay them $40,000 to reimburse them for the lost business from the broken cable. The letter threatened a lawsuit if Frank did not pay within thirty days. Frank immediately called the attorney on the telephone and admitted that he broke the cable and that he had not checked before digging to see if there was a cable. However, he argued that a $40,000 payment was unreasonable and he asked if they would agree not to sue if he paid them $5,000. Starr refused, and Starr ultimately sued Frank, seeking $40,000 in damages. At trial, Starr sought to admit the statement that Frank made over the phone to Starr's attorney in order to prove that Frank was liable in the case. Should the judge admit or preclude Frank's statement?

Preclude the entire statement

Police received an anonymous tip that cocaine was being sold out of an apartment. Officer Jordan went to the apartment undercover to attempt to purchase cocaine. Two other undercover police officers waited outside the apartment building and saw Officer Jordan enter. Officer Jordan stayed inside for five minutes, and then the officers saw him come back outside. The officers meant to join him, but he gave a signal that indicated he believed he was being followed. Instead, the officers met Officer Jordan back at the precinct two hours later. There Officer Jordan told them that he climbed two flights of stairs, knocked on the door of the designated apartment, and bought cocaine from a man with dark curly hair wearing a red leather jacket. The officers immediately returned to the apartment building to make the arrest, and as they climbed the flights of stairs, they saw a man with dark curly hair wearing a red leather jacket walking down the stairs. They arrested the man, whose name was Edward Heath. There were no narcotics found on Heath, and nothing connecting him to the apartment or the drug sale other than the statements of Officer Jordan. Heath was charged with sale of narcotics. Unfortunately, Officer Jordan was killed in a car crash one week before Heath went on trial. One of the officers who talked to Office Jordan at the precinct attempted to testify as to the description Officer Jordan provided about the person who sold him drugs. Heath objected to the testimony as hearsay. What should the judge do?

Preclude the evidence as hearsay.

Harold is suing Avery, claiming that Avery ran a red light in her car and struck him as he crossed the street in a crosswalk. Avery claims that the light was green when she entered the intersection and that Harold was crossing the street against the light. Harold wants to call Ronald, Avery's co-worker. If allowed to testify, Ronald will say that he has ridden with Avery hundreds of times and that Avery has a "habit of being a reckless driver." Ronald will support this claim by testifying that Avery routinely runs red lights, drives at speeds significantly over the speed limit, and is inattentive to the road. Harold objects to this testimony. Should the judge admit or preclude Ronald's testimony?

Preclude the evidence as improper propensity evidence.

Jonah is on trial for possession of cocaine. Jonah takes the stand in his own defense, claiming that the cocaine the police found in his car belonged to somebody else. Jonah has a prior conviction from five years ago for obstruction of justice, which is a felony. The elements of obstruction of justice are that an individual used violence or deception in order to impede a criminal investigation. The prosecutor seeks to admit Jonah's prior conviction to impeach him. What should the judge do?

Review the indictment from Jonah's obstruction of justice conviction to determine whether the acts he was charged with involved acts of dishonesty. If so, the crime is automatically admissible to impeach. If not, the judge should admit the prior conviction only if its probative value to impeach Jonah in this case outweighs the unfair prejudice to Jonah.

After reviewing Samantha's tax returns for the past three years, the Internal Revenue Service ("IRS") contacted Samantha and asked her to come to the local IRS office to explain some irregularities. The IRS agent suspected that Samantha had intentionally understated her income by over $100,000 and was hoping that he could elicit some incriminating statements from her during the meeting. At the meeting, the IRS agent presented Samantha with the evidence of the discrepancies between her actual income and the income she reported on her taxes, and he told her that she faced significant civil penalties. Samantha admitted that she made a mistake in calculating her taxes, but claimed it was merely an oversight. She offered to pay the minimum fine in order to avoid additional penalties. The IRS agent refused the offer, and the meeting ended. The IRS turned the case file over to the United States Attorney's office, and Samantha was ultimately criminally charged with income tax evasion. At trial, the government sought to admit her statements in which she confessed to understating her income and offered to pay the minimum fine. Samantha objects. Are Samantha's statements admissible?

Samantha's offer to pay the minimum civil penalty is inadmissible, but her statement admitting to understating her income is admissible.

Stuart is charged with stabbing a person in an alleyway outside a bar. He is pleading self-defense. During his defense case, his attorney has four character witnesses he wants to call. Which of them is most likely to be allowed to testify?

Stuart's father will testify that he has known Stuart his entire life and that Stuart has always been peaceful and never loses his temper.

Louis was stopped on the street pursuant to a legal Terry stop and the police officers frisked him and recovered a handgun. The officers ran his records and found that three years ago Louis had been convicted of burglary, a felony. Louis was charged with knowingly possessing a firearm while being a convicted felon. The elements of the crime are that the defendant (1) knowingly possessed a firearm; and (2) the defendant is a convicted felon. At trial, Louis admitted that he had been convicted of burglary three years ago, but he testified that he did not know that burglary was a felony. He will testify that he only received a six month sentence, and that he honestly believed that a crime was not a felony unless the defendant served a year or more in prison. The prosecutor objects to Louis' testimony. How should the court rule?

Sustain the objection because Louis' testimony is irrelevant.

The state is prosecuting Derek for stock fraud. Before the trial begins, Derek's attorney brings a motion in limine seeking to preclude the prosecutor from admitting evidence of Derek's two prior convictions, one for insurance fraud and one for insider trading. Both the prosecutor and Derek's attorney file motions on the issue. The judge ultimately rules that Derek's conviction for insider trading is admissible in the prosecutor's case-in-chief, but she withholds judgment on conviction for insurance fraud. During the prosecutor's case-in-chief, the prosecutor offers evidence of both the insider trading conviction and the insurance fraud conviction. At that time, Derek's attorney does not object to either conviction, and they are both admitted. Derek is convicted, and he is now appealing the conviction, arguing that the judge incorrectly admitted the two convictions. Is the appellate court allowed to review these two evidentiary rulings?

The appellate court can review the decision to admit the insider trading conviction, assuming the decision affected Derek's substantial rights, but it can only review the decision to admit the insurance fraud if its admission affected Derek's substantial rights and the trial judge's decision was a plain error.

Plaintiff is suing her former employer for breach of contract. During the trial, the defendant (plaintiff's former employer) asked the plaintiff on cross-examination how she lost the job that she had prior to starting work for the defendant. Plaintiff's attorney objected, but she did not give a reason for the objection, and her objection was overruled. The plaintiff testified that she had been fired from her previous job for drug use on the job. Plaintiff ultimately lost at trial, and is now appealing. She argues that the trial judge improperly allowed the defendant to ask about the reason she was fired from her previous job. What should the appellate court do?

The appellate court should affirm the judgment unless the trial court made a plain error and the error affected a substantial right of the plaintiff.

A plaintiff in a civil case offers hearsay evidence against the defendant. The defendant fails to object to the evidence at the time, and the evidence is admitted. After the plaintiff wins the case, the defendant appeals. One of the defendant's grounds for appeal is that the hearsay evidence was improperly admitted at trial. How should the appellate court proceed?

The appellate court should only reverse the decision if the admission of the hearsay evidence was an error that is clear and obvious under current law, that affects the defendant's substantial rights, and that would seriously affect the fairness and integrity of the judicial proceedings if left uncorrected.

Jaden, Inc, is suing Poseidon, Inc. for breach of contract. In response to Jaden's discovery request, Poseidon turned over approximately 12,000 e-mails that had been sent between Poseidon employees during the relevant time period. Poseidon did not review any of these e-mails; its Information Techology manager simply copied all of the e-mails from the relevant time period into a large data file and sent the data file to Jaden. Once Jaden began sifting through the e-mails, it found hundreds of e-mails from the Poseidon general counsel to other employees which are clearly privileged. Jaden notified Poseidon of the privileged e-mails, and Poseidon told Jaden to immediately delete all of the e-mails. Jaden filed a motion in court, claiming that when Poseidon disclosed the e-mails to Jaden, it implicitly waived the attorney-client privilege for those e-mails. Did Poseidon waive the attorney-client privilege?

The attorney-client privilege is waived because Poseidon did not take reasonable steps to prevent disclosure

On the morning of January 24th, Larry and his wife Ingrid got into an argument. Larry slapped Ingrid in the face and threatened to break her nose if she talked back to him again. Ingrid was very upset and began crying, but she did not call the police. Instead, she went to her job as a nurse and worked a full day at the hospital. At the end of her shift, another nurse noticed that she had a mark on her face and asked her what happened. She shook her head and said: "Larry hit me again. Next time he said he would break my nose. I really don't know what to do." Ingrid's co-worker was shocked at the news and convinced Ingrid to call the police. Larry was arrested and charged with assault and menacing. At trial, the prosecutor called Ingrid to testify about what happened. Ingrid took the stand, but then testified that Larry never hit her or threatened her. The prosecutor then called Ingrid's co-worker to testify about what Ingrid told her at the end of the day. The defense attorney objected to this testimony. Is the testimony of Ingrid's co-worker admissible?

The co-worker's testimony is inadmissible to prove the truth of the matter asserted, but it is admissible to impeach Ingrid

An oil tanker belonging to Davis Oil ruptured, and the resulting oil spill damaged the coastline of Florida and Georgia. The Environmental Protection Agency ("EPA") sent a team out to investigate the extent of the damage. The EPA team interviewed fishermen and coastal residents, and issued a report. Local businesses and fishermen are suing Davis Oil in a class action suit. They seek to admit parts of the EPA report as evidence against Davis. Which parts of the report (if any) are admissible?

The conclusion of the report, which states that "the damage to the local economy exceeds $300 million, and Davis Oil did nothing to mitigate this damage during the first three days after the oil spill."

Defendant Davidson Motors manufactures automobiles. On February 15th of this year, a motor in one of its cars spontaneously caught fire, causing property damage to the surrounding area. The plaintiff is the owner of the car and is now suing Davidson, alleging that the motor was defectively designed. It is undisputed that Davidson issued a recall letter on or about February 15th, ordering all of its customers to take their cars into a dealership for mechanical work that would fix the defect that caused the plaintiff's car to catch fire. There is no dispute that the recall letter, if it was issued after the plaintiff's motor caught fire, would be a subsequent remedial measure under Rule 407 and would therefore be inadmissible in the plaintiff's case if offered to prove liability. However, the plaintiff argues that the defendant issued the recall letter before the plaintiff's car caught fire in response to the defendant's own testing. The defendant argues that it issued the recall letter after the plaintiff's car caught fire in response to the plaintiff's accident. The defendant offers affidavits from its chief safety officer and its marketing director in which the affiants assert that the recall letters were issued on February 16th, after the defendant learned about the plaintiff's car. What is the proper procedure to determine the admissibility of the recall letter?

The decision of whether the recall letter was issued before or after the plaintiff's car caught fire is a question for the judge to decide, and the judge should do so outside the presence of the jury. The judge is allowed to consider the affidavits in making her decision, even though the affidavits are otherwise inadmissible hearsay.

Defendant Davidson Motors manufactures automobiles. On February 15th of this year, a motor in one of its cars spontaneously caught fire, causing property damage to the surrounding area. The plaintiff is the owner of the car and is now suing Davidson, alleging that the motor was defectively designed. It is undisputed that Davidson issued a recall letter on or about February 15th, ordering all of its customers to take their cars into a dealership for mechanical work that would fix the defect that caused the plaintiff's car to catch fire. There is no dispute that the recall letter, if it was issued after the plaintiff's motor caught fire, would be a subsequent remedial measure under Rule 407 and would therefore be inadmissible in the plaintiff's case if offered to prove liability. However, the plaintiff argues that the defendant issued the recall letter before the plaintiff's car caught fire in response to the defendant's own testing. The defendant argues that it issued the recall letter after the plaintiff's car caught fire in response to the plaintiff's accident. The defendant offers affidavits from its chief safety officer and its marketing director in which the affiants assert that the recall letters were issued on February 16th, after the defendant learned about the plaintiff's car. What is the proper procedure to determine the admissibility of the recall letter?The decision of whether the recall letter was issued before or after the plaintiff's car caught fire is a question for the judge to decide, and the judge should do so outside the presence of the jury. The judge is allowed to consider the affidavits in making her decision, even though the affidavits are otherwise inadmissible hearsay.

The decision of whether the recall letter was issued before or after the plaintiff's car caught fire is a question for the judge to decide, and the judge should do so outside the presence of the jury. The judge is allowed to consider the affidavits in making her decision, even though the affidavits are otherwise inadmissible hearsay.

The plaintiff in a civil case called Oliver to testify. On cross-examination, the defendant's attorney asked Oliver if it was true that on September 15th of the previous year, he filed a fraudulent claim for insurance. Oliver denied that he did so. The defendant's attorney then admitted an affidavit signed by Oliver in which he admitted to the insurance company that his September 15th claim was fraudulent. The defendant's attorney admitted the affidavit to prove not only that Oliver filed the fraudulent claim (and thus had a propensity for dishonesty) but also to prove that Oliver had just lied to the jury when he denied filing it. Assume the plaintiff objected to all of the defendant's questions and actions during this time. At what point (if any) should the judge have sustained the plaintiff's objections and cut off this line of inquiry?

The defendant should not have been allowed to admit the affidavit, since it is extrinsic evidence.

Winslow Laboratories is suing Hanover, Inc. for patent infringement. As part of the discovery process, Winslow deposed five of Hanover's employees, including Hanover's Vice-President in charge of Research and Development. Later, the Vice-President also submitted a sworn affidavit relating to the case. At trial, Hanover decided not to call the Vice-President as a witness because Hanover's lawyers were worried that he would not seem credible to the jury. Instead, Hanover sought to admit the Vice-President's deposition testimony and his affidavit. The evidence is being offered for the truth of the matter asserted. Winslow objects to this as hearsay. Should the judge admit or preclude the Vice-President's deposition testimony and affidavit?

The deposition and affidavit are hearsay and should be precluded.

Agents from the Drug Enforcement Agency got an anonymous tip that someone was selling drugs out of a black car parked outside Happy Times bar. When they arrived at the Happy Times bar, they saw a black car in the parking lot. As they were waiting outside the bar, a woman came out of the bar and opened up the trunk of the car. As soon as she opened the trunk, one of the agents approached her and showed her his badge. When the woman saw the badge, she immediately took off running back into the bar. One of the agents caught her, and another one looked inside the open trunk and found a paper bag with ten small baggies of cocaine inside. The woman was arrested and identified as Phoebe Gates. She was charged with knowingly possessing cocaine. At trial, the DEA agent wanted to testify about the fact that Phoebe ran as soon as she saw the law enforcement badge. Phoebe's attorney objected. Is the fact that Phoebe ran admissible?

The fact that Phoebe ran is admissible because it tends to prove she knew there was cocaine in the trunk

Simon is on trial for stealing a car. The car in question was a 2005 Honda Accord. In order to prove grand larceny, the prosecutor has to prove that the value of the item stolen is greater than $5,000. The prosecutor had the owner describe the car, and she said the car had 120,000 miles on it and that its paint was chipped and peeling. The prosecutor then called Jack, a used-car salesman. Jack has never had formal training as an automobile appraiser, but he has been in charge of setting prices for his dealership for twenty-five years. Jack had never seen the car in question, nor had he seen a picture of it or heard the owner describe it. The prosecutor asked Jack how much a 2005 Honda Accord would be worth, assuming it had 120,000 miles on it and that its paint was chipped and peeling. Jack replied that it would be worth between $4,000 and $5,000. The prosecutor then asked Jack how much it would be worth assuming it had only 50,000 miles on it and that its paint was perfect. Jack replied that it would be worth $10,000. What, if anything, is wrong with Jack's testimony?

The first question to Jack is appropriate, but the second question to Jack should probably not be allowed.

Rick owned a home near a river on a flood plain, and one year after many days of rain, the river rose and flooded his house, causing severe damage. Rick contacted his insurance company, but the insurance company refused to pay anything for the damage. Rick ultimately sued the insurance company for the damages he suffered. The insurance contract that Rick and the company signed was ambiguous as to whether the policy covered flood damage for houses inside the flood plain. Under the relevant contract law, the oral representations of the parties that were made when signing the contract are relevant when a court is interpreting the meaning of the contractual provisions. Rick testified that the insurance agent never told him that flood damage was not covered. The defendant insurance company called the agent who sold Rick the policy. The agent testified that he honestly did not remember what he said when he sold Rick the policy, since he sold the policy ten years ago. The agent will testify that every time he sells a policy to a homeowner whose home is in the flood plain, he notifies the homeowner that the basic policy does not cover flood damage. He estimates that he has sold hundreds of these policies during his career as an insurance salesman. Rick objects to this testimony. Is the insurance agent's testimony admissible?

The insurance agent's testimony is admissible as habit evidence.

Bernard Stansfield's wife mysteriously disappeared, and police suspected that Stansfield had killed her and hidden the body. Two days after the disappearance, Stansfield called his friend Harry Gerson and talked to him on the phone for three hours. Right after the phone call was over, Stansfield bought a one-way plane ticket to Australia for that evening and drove to the airport. The police apprehended him just before he boarded the plane. Stansfield is now on trial for murdering his wife. The police seek to call Gerson to the stand to force him to testify about what Stansfield told him during their three-hour conversation. Stansfield files a motion in limine objecting to this testimony, arguing that Gerson is a trained psychologist and was providing psychological counseling to Stansfield during the course of the conversation. Thus, Stansfield argues that the conversation is privileged under the psychotherapist-patient privilege. Stansfield offers an affidavit signed by Gerson in which he describes his education and training as a psychologist and states that the sole purpose of the conversation was to counsel Stansfield. The prosecutor argues that Stansfield and Gerson were only talking as friends. She wishes to call Gerson to the stand in a preliminary hearing, and force him to testify about what the two of them discussed in order to determine whether Gerson was providing psychological counseling at the time. What evidence is the judge allowed to consider when deciding Stansfield's motion in limine?

The judge may consider the affidavit, but may not require Gerson to testify.

Gail is an eyewitness in a hit-and-run prosecution. When the prosecutor asks her for the license plate of the car that she saw strike the pedestrian, she states that she cannot remember the license plate number. The prosecutor asks if seeing a photo of the license plate would refresh her recollection, and Gail says yes. The prosecutor has a photo of the license plate. What is the proper procedure to refresh the witness' recollection?

The witness should look at the photo and then it should be taken away from her and she should testify about the license plate number from her refreshed recollection.

Debra is on trial for shooting her husband in the back while they were out hunting together. Debra admits that she shot him, but claims that the shooting was accidental; she was shooting at a deer and did not see her husband in front of her. The prosecution wants to admit evidence that one week before the shooting, Debra attempted to poison her husband by placing rat poison in his morning coffee. Her husband did not drink the coffee, but the housekeeper found the undrunk cup and will testify that it "smelled bitter, like the poison I set out for the rats." The housekeeper noticed that the box of rat poison had been moved that morning by someone other than herself. The housekeeper will also testify that Debra always made the coffee for herself and her husband. Should the judge admit the housekeeper's testimony?

The judge should admit the housekeeper's testimony if the judge believes that a reasonable jury could find by a preponderance of the evidence that Debra put rat poison in the coffee.

Levi is charged with raping Irene after the two of them went out on a date. Levi and Irene lived in the same college dormitory at the time. Levi claims he believed the sex was consensual; Irene testifies that she clearly indicated she did not want to engage in sexual intercourse and attempted to push Levi away, but was unable to prevent the act. As part of her case-in-chief, the prosecutor wants to call Kristine, who will testify that Levi sexually assaulted her after a date in high school six years earlier. Kristine reported the crime to the police, but she says the police did not take her seriously and no charges were ever filed. For his part, Levi seeks to call Nancy, who lived in the same dorm as Irene and Levi. Nancy will testify that everybody in the dorm knew Irene's reputation, and that she was someone who would always consent to sex after a date, and would usually pretend to protest because "that made it more fun for her." Should the judge allow the testimony from Kristine and/or Nancy?

The judge should allow Kristine to testify to prove Levi's propensity to engage in violent sexual acts against women (subject to a Rule 403 analysis), but should preclude Nancy's testimony.

Thomas is suing PharmaGiant, a large drug company, arguing that PharmaGiant's new anti-cancer drug causes permanent memory loss and other brain damage in patients. As part of his case-in-chief, Thomas calls Dr. Kalas, who is an expert in epidemiology. Dr. Kalas testifies that he relied on data on brain damage gathered by independent doctors around the country and then processed the data to determine that brain damage was five times more common in patients who took PharmaGiant's new drug. PharmaGiant objects to this testimony, arguing that Dr. Kalas did not personally test the patients for brain damage; he relied on other doctors to measure the brain damage and then report the results to him. All parties agree that this data would be inadmissible for the truth of matter asserted under the hearsay rule. Thus, PharmaGiant argues that Dr. Kalas should not be able to testify about his conclusions. Should the court allow Dr. Kalas to testify about the underlying data and his conclusions?

The judge should determine whether the data gathered by the independent doctors on brain damage is of a type reasonably relied upon by experts in the field of epidemiology; if it is, the judge should allow Dr. Kalas to testify about his conclusions. Dr, Kalas can testify about the underlying data only if the judge determines that the probative value in helping the jury evaluate the opinion substantially outweighs the prejudicial effect of admitting the data.

Michael is on trial for murder. The prosecutor's theory of the case is that Michael stabbed his roommate six times in the chest while his roommate was sleeping. The prosecutor wishes to admit three photographs of the crime scene as it appeared when officers first arrived, showing the victim in the bed, with the stab wounds clearly visible and blood soaking through his sheets and nightclothes. Michael's attorney objects to the photos, arguing that they will unfairly prejudice the jury. Should the judge preclude the photos?

The judge should preclude the evidence if the risk of unfair prejudice substantially outweighs the probative value of the evidence.

Oscar was applying for citizenship, and he hired Victoria as a lawyer to help him prepare his application. First, Victoria interviewed Oscar for twenty minutes about certain sensitive issues that might prevent him from becoming a citizen. During this interview, Greg, her paralegal, was present and taking notes. Greg then left to type up the notes and put them in Oscar's file. Then Victoria told Oscar that part of the process for becoming a citizen was to fill out an application which had to be submitted to the United States government. She gave him the application and he filled it out and then gave it back to her. She told him she would file the application by the end of the week. The next day the federal immigration authorities showed up at Victoria's office. They said that they were investigating Oscar's immigration status to determine whether he was legally in the country. They came with a court subpoena demanding all the notes of the interview between Victoria and Oscar, and demanding a copy of the application that Oscar filled out in order to become a citizen. Victoria refused to hand over any of the information, and the immigration officers sued her, demanding that she turn it over. Are the notes from the interview and Oscar's application protected by attorney-client privilege?

The notes from the interview are privileged, but the application is not, because it was not part of a confidential communication between the client and the attorney.

Over the past few months, James has had fantasies of sexually assaulting and hurting young girls. Last week, he acted on the fantasies for the first time, and attacked a girl he saw on a playground. He has been very disturbed by his fantasies and his actions, and so he immediately went to Debra, his social worker, and asked if he could speak to her about his mental issues. Debra is a licensed social worker, but not a licensed psychologist or psychiatrist. However, she agreed to speak with him, promising that the conversation would remain confidential and that she would do whatever she could to provide him with counseling. During the conversation James confessed to his crime and said he was afraid he might do the same thing again. Debra was shocked at what she heard and—without James' consent—went straight to the police. The police arrested James on charges of child molestation. At James' trial, the prosecutor seeks to call Debra to testify about what James told her. Debra is willing to testify, but James objects, arguing that the conversation was privileged. How should the court rule?

The privilege applies and Debra should be precluded from testifying.

Wilbur was stabbed in the chest outside of a bar late one night. The police responded to the call within minutes, and found Wilbur breathing heavily, crying, and in a lot of pain. Wilbur said: "A man named Hank stuck me with a knife! He is wearing a red sweatshirt and has short black hair. He ran south down the street." At the time of Wilbur's statement, Hank was still at large and, as far as Wilbur and the police knew, still carrying a knife. The police located Hank Johnson hiding in an alley down the street and arrested him. He was wearing a red sweatshirt and had short black hair. At Hank's trial, the prosecutor called Wilbur, but he refused to come to court. Instead, the prosecutor called the police officer who responded to the scene and asked him to testify about what Wilbur told him. Hank objected, arguing that Wilbur's statement was hearsay. Is Wilbur's statement admissible?

The statement is admissible as an excited utterance and it does not violate the Confrontation Clause.

Peter is suing Dr. Denard for malpractice. Peter had suffered from appendicitis, and Dr. Denard was the doctor who removed his appendix. Unfortunately, Dr. Denard left a surgical sponge inside of Peter's body, which resulted in severe complications. At trial, Dr. Denard claims that, consistent with standard medical practice, he relied on his nurse during the operation to keep track of how many sponges he had used. He testifies that as the operation was ending, he asked his nurse how many sponges he had put into Peter's body, and she had said "four." In fact, there had been five. Dr. Denard removed four sponges and closed the wound, inadvertently leaving a sponge inside of him. Peter objects to the part of Dr. Denard's testimony in which he testifies that the nurse answered "four" in response to his question. Peter argues this statement should be barred by the hearsay rule. The nurse has not testified in this case. Is Dr. Denard's statement admissible?

The statement is admissible, because it is not being offered for the truth of the matter asserted.

Nadine was shot in the chest in her home. She was found by her sister two hours later, barely conscious. Her sister asked her what happened, and she answered in a calm, clear voice: "My boyfriend Chris shot me," and then she lost consciousness. The police located Chris Dawkins, her boyfriend, and arrested him. Nadine is now in a coma. Chris is being prosecuted for attempted murder. The prosecutor calls Nadine's sister to the stand to testify about what Nadine said. Chris objects, arguing that Nadine's statement is inadmissible hearsay. What is the proper ruling?

The statement is inadmissible hearsay.

Peter is suing Downing Lawn Vehicles. Peter claims that he was using a model 6100 Downing riding mower on his lawn, and when he turned the speed to maximum, one of the front wheels detached from the axle, causing the riding mower to crash into a tree and throwing Peter from the mower. Peter claims that these mowers suffer from a design defect which causes the front wheels to occasionally detach from the axle when the mower is driven at a high speed. As evidence to prove his allegation, Peter wants to admit the testimony of three other individuals who used a model 6100 Downing riding mower the year before Peter had his accident. In each case the front wheel fell off the axle when the owners drove them at maximum speed. All three of these individuals will also testify that they wrote Downing a letter complaining of the incident immediately afterwards, and Peter will seek to admit the letters that they wrote. What is the proper ruling?

The testimony is admissible to prove that this model riding mower had a design defect and the letters are admissible to prove that Downing had notice of the defect.

Ronald is a truck driver who works as an independent contractor, hauling freight for small businesses across the country. A recycling center in California contacted Ronald and told him it would pay him a set fee if he transported ten truckloads of aluminum cans from their warehouse near San Francisco to the processing plant near Los Angeles. Ronald transported all ten truckloads, and the recycling center paid him $10,000. Ronald claims that the recycling center had promised to pay him $15,000, and he sued them for the difference. At trial, the president of the recycling center takes the stand and testifies: "When we first contacted Ronald, I told him we would pay him $10,000, to haul ten loads." Ronald objects this testimony as hearsay. Is the president's testimony admissible?

The testimony is not hearsay, because it is not offered for the truth of the matter asserted, but instead to prove that an offer with specific terms was made.

At trial, a party wishes to prove the following two facts: (i) Drinking alcohol can result in a state of intoxication; (ii) Individuals who are between the ages of eighteen and twenty drink alcoholic beverage at a greater rate than any other age group. What are the permissible ways to get these facts into evidence?

The trial court should take judicial notice of (i), but it should require the party to prove (ii) using expert evidence.

Ross was at work using a power saw that had been manufactured by Cutter Tools. Ross held the trigger of the saw steady with his left hand and used his right hand to guide the wood through the rotating blade. When the wood was cut, Ross leaned down to pick up the cut piece of wood and took his left hand off the trigger of the saw. However, the saw blade did not stop rotating, and as Ross leaned over to pick up the wood, his right hand came into contact with the saw blade and was severed from his arm. Ross sued Cutter Tools, alleging that their design was unreasonably dangerous because the saw blade did not immediately stop rotating when the user took his hand off the trigger. Which of the following is MOST likely to be admissible?

Two weeks after the incident, the same model of the Cutter Tools power saw severed the finger of a woman using it in her home when the blade kept rotating after she took her finger off the trigger.

One day Debra, a four-year-old girl, disappeared from her front yard where she had been playing. A few hours later her family got a ransom demand from her kidnapper. After three days of negotiations, Debra was released in a nearby park in exchange for $10,000. Police officers investigating the crime developed probable cause to believe that Greg was the kidnapper. They arrested Greg and he was tried for kidnapping. Greg's defense was mistaken identity; he claimed he was not the kidnapper. The prosecutor did not want to put Debra on the stand because of her young age. However, Debra described to her father the location where she had been held for three days. She described a basement with a fish tank with large red fish, a Superman poster on the wall, and bright blue carpeting. The prosecutor wants to call the father to the stand to testify that Debra described the basement this way. The prosecutor will then call a police officer who will testify that he searched Greg's home, and that Greg has a basement with a fish tank with large red fish, a Superman poster on the wall, and bright blue carpeting. Should the judge allow the father to testify about what Debra told him?

Yes, because Debra's statements to the father are not hearsay.

A police officer pulled over Sam's car after seeing it swerve numerous times into the oncoming lane of traffic. The officer ordered Sam out of the car and told him to take a Breathalyzer test. Sam refused. Based on Sam's erratic driving and Sam's refusal to take the test, the officer arrested Sam for driving while under the influence of alcohol. A person is guilty of this crime if his ability to drive a car is impaired because of consumption of alcohol. At trial, the prosecutor wants the officer to testify that Sam refused to take the Breathalyzer test. Is this evidence relevant?

Yes, because his refusal to take the test makes it more probable that he was in fact intoxicated.

Oswald is suing his former employer, Overton Power, in federal court. Oswald claims that after he worked at Overton's nuclear power plant for eight years, he developed cataracts in his eyes as a result of the high radiation levels at the plant. Overton states that there is no evidence that Oswald's cataracts were caused by high levels of radiation. In his case-in-chief, Oswald seeks to call Dr. Conway, an ophthalmologist who examined Oswald's eyes. Dr. Conway will testify that he was able to determine that Oswald's cataracts were caused by radiation after an intensive visual examination. Oswald objects, arguing that there is no way to determine whether cataracts are caused by radiation without a biopsy, in which a small piece of the cataract is sliced off and tested. The trial judge holds a pre-trial hearing, at which both Dr. Conway and Overton's experts testify. After the hearing, the judge is convinced that Dr. Conway's method of diagnosing cataracts is reliable, because it has been tested and has low error rates and has undergone some peer review. However, the judge also determines that Dr. Conway's method is not generally accepted by the scientific community and there are no standards for this method. Should the trial judge admit Dr. Conway's testimony?

Yes, because the federal courts apply the Daubert standard and this method passes the Daubert standard.

Gary was mugged on the street by two men with guns. He handed over his wallet and they ran away with it. Seconds later, in a state of great agitation, he called the police and reported the crime. In response to the questioning from the 911 operator, he told the police his location, a description of each of the robbers, and the direction in which they ran. The police caught the robbers ten minutes later and recovered Gary's wallet. The two suspects were charged with second degree robbery. A few weeks before trial, Gary was hit by a bus and killed. At trial, the prosecutor sought to admit his statements during the 911 call. Assume Gary's statements are properly authenticated. Are they admissible?

Yes, because they are excited utterances and they do not violate the Confrontation Clause because of the they are not testimonial if made during on ongoing emergency.

The police exercised a search warrant on Carl and Stacy's apartment and they found half a kilo of cocaine hidden under the couch in a guest room. Carl and Stacy were both arrested. After the arrest, Carl maintained his innocence, claiming that he had no idea there was cocaine in the apartment. For her part, Stacy immediately told the police that the cocaine was hers and that Carl didn't know anything about it. The prosecutor did not believe that Carl was innocent, and she charged both Stacy and Carl with possession of a controlled substance with the intent to sell. Stacy pled guilty, but Carl went to trial. At trial, Carl took the stand and testified that he knew nothing about the cocaine, but that he had seen Stacy hiding something under the couch in the bedroom. He will further testify that she had told him not to worry about what it was and never to look at it, an order which he obeyed. In her rebuttal, the prosecutor offered evidence that Stacy had only a sixth grade education, that she had a very low I.Q., that she immigrated to this country only eighteen months before the police raided the apartment, and that she speaks almost no English. Carl objected to this evidence and was overruled. After the prosecutor's rebuttal, Carl responded by offering evidence that Stacy has two prior convictions for selling cocaine over the past eighteen months. The prosecutor objected to this and was overruled. Stacy did not testify in this case. Was the judge correct in making her rulings?

Yes, both pieces of evidence were properly admitted.

Betsy is suing her doctor, arguing that the doctor did not exercise the required level of care when he operated on her knee. Her husband testifies in her case, describing the difficulty that Betsy had moving around in the weeks after the surgery. At one point during his testimony, he testifies that "two months after the doctor performed the operation, Betsy could still not walk up the stairs of our home." The doctor's attorney cross-examines the husband and asks him whether it is true that two months after the operation, he had dinner with a friend and he said: "Betsy is really recovering well after her surgery. She can go up and down the stairs at our house with no problem." Can the doctor's attorney ask her this question?

Yes, but Betsy can request a limiting instruction which tells the jury to only consider the prior statement for impeachment purposes and not for the truth of the matter asserted.

Draco Chemical's pipeline burst and spilled toxic chemicals into a residential neighborhood. Two separate homeowners, Thomas and Darlene, sued Draco Chemical for damages arising from the spill. The cases were severed and proceeded to trial separately. In the first case, when Thomas was the plaintiff, Draco called its chief engineer to the stand to testify. After the first case was over, the chief engineer was hit by a car and died. The court is now trying the second case, in which Darlene is suing Draco. Draco wants to admit the testimony of its chief engineer as evidence in its defense. Is the chief engineer's testimony admissible?

Yes, but only if Thomas had a similar motive to cross-examine the chief engineer in his case as Darlene does now in her case.

Frank is being prosecuted for armed robbery after allegedly holding up Debra in the street at gunpoint and stealing from her. The prosecutor calls Debra to the stand and asks her to tell the jury what was stolen from her during the incident. Debra says: "The guy took my watch and my wallet." The defense attorney has a copy of the police report, filled out by Officer Gendry, the officer who responded to the scene. On the report Officer Gendry wrote: "I spoke to Debra, the victim, while I was on the scene, and Debra told me that the perpetrator stole her wallet. I asked if the perpetrator took anything else, and Debra said no." On cross-examination, can the defense attorney ask Debra about the statement she made to the police officer in which she allegedly said that only her wallet and nothing else was taken? The defense attorney is offering this evidence to impeach Debra, not for the truth of the matter asserted.

Yes, the defense attorney can ask Debra about the statement she made to the police.

Alex is on trial for burglary. The house he allegedly broke into had been for sale, and the prosecutor's theory is that a week before the burglary, Alex had shown up at the house as a potential buyer in order to learn the lay of the house and where it was vulnerable to break-ins. The defendant admits he came to look at the house as a potential buyer, but claims he was nowhere near the house when it was burgled. Alex has two prior convictions. One is for a felony burglary that occurred six years ago, in which he broke into a house that was for sale after he posed as a buyer and walked through the house three days before the crime. The other conviction is a misdemeanor for filing a false police report two years ago. Can she admit evidence of these prior crimes?

Yes, the felony burglary is admissible (subject to Rule 403) whether or not Alex testifies, but the filing of the false police report is admissible only if Alex testifies.

Sarah's boyfriend George was killed in his own kitchen with a knife early one morning. He had a knife in his hand. When the police responded to the call, they found Sarah in the kitchen crying, and the back door to the house was open. When the police asked Sarah what happened, she said nothing. The police continued their investigation over the next two weeks, and came to the conclusion that Sarah's grown son Andrew had killed George. The police arrested Andrew and charged him with murder. At trial, Andrew called his mother Sarah as a witness. She testified that she was the one who killed George, and that she acted in self-defense. She testified that George had come home drunk and began arguing with her and then grabbed a knife and swung it at her chest, and that she was able to grab her own knife and stab him in the chest. She also testified that she washed the knife she used and put it back in the drawer before the police arrived. She further testified that her son Andrew was never present that night. On cross-examination, the prosecutor asked Sarah if it was true that she said nothing when the police originally asked her what happened. Is this question permissible?

Yes, the question is admissible to impeach Sarah because silence in this context is inconsistent with her testimony of self-defense, since it would be natural for Sarah to have told the police about what happened when asked earlier.

Jackson was pulled over after he was weaving across lanes of traffic. After observing him for a few minutes, the officer who pulled him over came to the conclusion that Jackson was drunk. Jackson refused to take a Breathalyzer test. The officer arrested Jackson for driving while intoxicated. After the arrest, the officer searched the car and found an empty bottle of wine and an empty paper bag. Inside the bag was an automated receipt from a liquor store that recorded a purchase of a bottle of wine with a credit card that matched Jackson's credit card number. The time of purchase listed on the receipt was 9:30 PM on that night. Jackson had been pulled over at 10:30 PM. At trial, the prosecutor wants to admit the receipt from the store to prove that Jackson bought the wine at 9:30 PM. Is the receipt admissible?

Yes, the receipt is admissible, but the prosecutor must first demonstrate that the machine that generated the receipt is reliable.

Blackstone is a company which manufactures smart phones. Jonathon purchased a Blackstone phone and used it for two years, when he was diagnosed with brain cancer. He sued Blackstone, claiming that the radio waves emanating from his phone caused his cancer. In his case-in-chief, Jonathon calls Dr. Dark, who will testify that in his expert opinion the Blackstone phone can cause brain cancer. Dr. Dark has conducted numerous studies which have backed this up, and he has also spoken with dozens of other cancer specialists across the country, all of whom have reported a sharp increase in the instances of brain cancer in their patients who use Blackstone telephones. Dr. Dark testifies that in determining causation, doctors routinely rely on reports of correlation as reported by other doctors. Blackstone concedes that Dr. Dark is an expert who can give his opinion, but Blackstone objects to his testimony that dozens of other cancer specialists have reported the sharp increase in cancer for users of Blackstone telephones. Should Dr. Dark be permitted to testify about his conversations with the other specialists?

Yes, the statements are admissible, but only to assist the jury in evaluating Dr. Dark's expert opinion, and only if the trial judge concludes that the probative value in assisting the jury to evaluate his opinion substantially outweighs their prejudicial effect.

As part of a sting operation, the Capital City police department sent out an undercover officer to sell cocaine on the street. Thomas was walking by the undercover officer when the officer called him over and convinced him to buy some cocaine. After Thomas bought the cocaine, he was immediately arrested for possessing a controlled substance. At trial, Thomas is using an entrapment defense. Under the laws of his jurisdiction, this defense requires him to prove that he did not have any predisposition to commit the crime, and that the law enforcement officer persuaded him to take possession of the drug. In his defense, his calls his friend Terry. Terry will testify that a year ago he was at Thomas' house for a party, and Thomas noticed a number of guests snorting cocaine in the corner. Terry will further testify that Thomas became very angry, walked over to the group, and told them to stop and to never use cocaine in his house again. The prosecutor objects to Terry's testimony. Is it admissible

Yes, the testimony is admissible.

Greg was suffering from depression, and his doctor proscribed Simtec, a new antidepressant developed by HealthFirst Pharmaceuticals. One year later, Greg had developed stomach cancer, and he sued HealthFirst, arguing that Simtec caused his stomach cancer. As part of his case, he calls his personal physician, Dr. Fleming, who diagnosed Greg's stomach cancer and predicts that Greg only has a few months to live. Dr. Fleming bases his opinion on a series of tests that hospital staff conducted on Greg at Dr. Fleming's request. Dr. Fleming himself did not conduct these tests, but he and other doctors routinely rely upon these sorts of tests in making a diagnosis, and the hospital routinely conducts these types of tests. Members of the hospital staff who conducted the test are not available to testify in the case. Dr. Fleming will testify on direct that in his expert opinion Greg has stomach cancer, but he will not discuss the tests that he reviewed or in any way explain the basis of his decision. The trial judge allows this. On cross-examination, HealthFirst seeks to admit the tests that Dr. Fleming relied upon, in order to show that the tests were flawed and insufficient to reach a diagnosis of stomach cancer, much less a prognosis of only a few months to live. The trial judge allows this as well. Was the trial judge correct?

Yes, the trial judge properly admitted both Dr. Fleming's opinion without any underlying data on direct, and the trial judge properly allowed HeathFirst to admit the underlying data on cross.

Egbert Oil runs a pipeline from the Gulf of Mexico into the Rocky Mountain region. On June 15th, the pipeline ruptured, spewing thousands of gallons of oil into the farmlands of eastern Colorado. The Colorado Environmental Agency ("CEA") immediately mobilized to contain the spill, and generated thousands of pages of mandated reports describing the spill and the damage done to the farmland. The CEA had never before responded to an oil spill; its usual purview was dealing with forest fires and pollution from factories; however, its statutory mandate included a requirement that it record and remediate any damage from oil spills. Dozens of landowners in Colorado are now suing Egbert, seeking restitution for the damage to their property. As part of their case, they are offering the CEA reports to prove the damage that was done to the land. Egbert objects to these reports as hearsay. Are the reports admissible?

Yes, they are admissible as public records

Stanley is accused of shooting and killing his wife. Stanley admits that he shot his wife, but claimed that it was an accident—that he had picked up the gun to see whether it was loaded and it went off. The prosecutor wishes to call Greg as one of her witnesses. Greg is one of Stanley's friends from the local bar, and Greg will testify that two weeks before Stanley shot his wife, Stanley offered Greg $1,000 if Greg would break into Stanley's house and kill his wife. Greg refused the offer, but never went to the police with the information until after Stanley's wife was killed. Stanley has not been charged with a crime for his offer to Greg. Can Greg testify about Stanley's offer?

Yes, to prove intent and lack of accident.

Pursuant to a valid court order, the FBI installed a wiretap on a telephone in a warehouse which they suspected was being used to store stolen property. The FBI recorded over fifty hours of conversation between the owner of the warehouse and another man who was referred to only as "Slick." During the course of the conversation, the warehouse owner mentioned a number of stolen items and Slick made offers to purchase them. Other evidence led the FBI to believe that "Slick" was actually Sam Gordon, a known criminal who sold stolen goods over the internet. The FBI agents running the wiretap called in Special Agent Donovan, who had used Gordon as an informant numerous times and was familiar with his voice. Donovan confirmed that the voice of Slick on the tape belonged to Sam Gordon. Gordon was ultimately arrested and charged with receiving stolen property. At trial, the prosecutor seeks to play some portions of the tape to the jury. The prosecutor intends on authenticating the tape by calling Special Agent Donovan to testify that the voice of the man called Slick belongs to Sam Gordon. Gordon will testify that the voice on the tape does not belong to him. Should the tape be admitted into evidence?

Yes. Agent Donovan's testimony is sufficient to authenticate the voice as Gordon's.

Corrine is suing Easy Cruisers, Inc. for a design defect in a yacht engine that Easy Cruisers built and sold to Corrine. During her case-in-chief, Corrine calls Karl to the stand as an expert witness. Karl is a high-school dropout with no formal training in boat design or mechanics. However, he runs his own marine engine repair shop, and has been personally fixing and rebuilding marine engines for over twenty-five years. Over the course of his career, he has repaired over a thousand marine engines for yachts. He will testify that he worked on Corrine's yacht engine after it broke down, and that in his expert opinion, the engine overheated and burned out most of the machinery because the oil line was improperly installed. Easy Cruisers objects to this testimony. Should the judge allow Karl's testimony?

Yes. Allow the testimony.

Debra is on trial for embezzlement after allegedly transferring six thousand dollars from a company account into her own personal account. The prosecutor has evidence that two years earlier, Debra stole two hundred dollars from her last employer by endorsing a check that was meant for the employer and depositing it into her own account. Will the prosecutor be able to admit this evidence?

Yes. If Debra calls a character witness who testifies that Debra has an honest character and is not the type of person who would steal, then the prosecutor can ask the character witness about the prior embezzlement on cross-examination of the character witness.

Jerry is suing his former employer, Galaxy Demolition, Inc. Jerry's job at Galaxy was to help tear down abandoned buildings and cart the waste to the local dump. Jerry contends that Galaxy assigned him to tear down an old building with asbestos inside the walls, and that Galaxy did not warn him about the asbestos or give him any protective equipment. Galaxy's defense is that any asbestos that Jerry might have been exposed to was a negligible amount and could not have caused any harm to Jerry. In his case-in-chief, Jerry wants to call his doctor, Dr. Walsh, to the stand. Dr. Walsh will testify that Jerry came to his office on a certain date and said he was having trouble breathing and that he was coughing up blood. Dr. Walsh will tell the jury that he referred Jerry to a specialist named Dr. Greenbaum. Unfortunately Dr. Greenbaum is not available to testify, but Jerry will testify that when he visited Dr. Greenbaum, the doctor performed some tests and then told Jerry that Jerry suffered from asbestos poisoning. Are there any hearsay problems with Dr. Walsh's testimony or Jerry's testimony?

Yes. Jerry should not be permitted to testify about what Dr. Greenbaum told him, since that is hearsay.

Gary was bicycling through an intersection when a pickup truck driven by Karen struck him and killed him instantly. Wendy was in the passenger seat of Karen's car and observed the entire incident. Ten minutes after the event, the police interviewed Wendy, who was crying and visibly upset. Wendy told the police: "I don't know what the bicyclist was thinking. We had a green light and he just barreled across the intersection right in front of us!" Gary's family is now suing Karen. Wendy is out of the country on business, so Karen calls the police officer to testify as to what Wendy said about the light being green for Karen. The judge admits this evidence over Gary's objection. In his rebuttal case, Gary calls Wendy's friend Veronica. Veronica will testify that (1) Wendy and Karen are best friends, and have known each other since childhood; and (2) one week after the event, Wendy told Veronica that Karen ran a red light and killed a bicyclist. The judge allows this testimony over Karen's objection. Should the judge have admitted either the police officer's testimony or Veronica's testimony?

Yes. The police officer's testimony about what Wendy said is admissible, and all of Veronica's testimony is also admissible.

Sarah is a minister in the High Church of Christ, a splinter Christian sect that was founded five years ago but is growing in popularity in numerous towns in the Midwest. Two months ago Justin, one of her parishoners, came to visit Sarah at her house and spoke with her privately for five hours about criminal acts he had committed against members of his family. Justin was arrested the next day. The police know that he visited Sarah the day before his arrest, and so the prosecutor has subpoenaed Sarah to testify in front of the grand jury. Sarah refuses, claiming that the communications with Justin were privileged. The prosecutor has come to the court to ask for a court order that compels Sarah to testify. Is the communication privileged?

Yes—as long as Sarah claims that Justin came to her seeking spiritual advice, the conversation is privileged.


Conjuntos de estudio relacionados

CS 109 Chapeter 9 Stacks & Queues for Data Structures.

View Set

chapter 10 motivation, personality, and emotions

View Set

Psych Ch 16, Psychology Chapter 15, Psychology

View Set

Intermediate Accounting Midterm 2

View Set

CH. 10 PLANETARY ATMOSPHERES... HAHA

View Set

Chapter 7: Long-Term Memory - Structure (Encoding, Retrieval, and Consolidation)

View Set

Chapter 18 - Formation of Sales and Lease Contracts

View Set

Chapter 12: The Future of Data Systems

View Set

Mrs. Long AP Psych Semester 1 Final- Multiple Choice Q&A

View Set